#14 Rosh Review

Réussis tes devoirs et examens dès maintenant avec Quizwiz!

Question: Which eye tumors present with hyphema?

Answer: Retinoblastoma and melanoma of the iris. Rapid Review Hyphema Patient with a history of blunt or penetrating trauma Complaining of blurry vision PE will show unequal pupils, injected conjunctiva/sclera and blood in anterior chamber Treatment is eye protection and rest with the head of the bed at 30 degrees all the times

Question: What is the most common cause of mitral stenosis?

Answer: Rheumatic heart disease. Rapid Review Mitral Stenosis Most common cause: rheumatic heart disease Dyspnea on exertion, hemoptysis Opening snap, diastolic apical murmur Atrial fibrillation

Question: Does the serum CK level predict the risk of acute renal failure?

Answer: No. CK level correlates well with degree of muscle injury but does not predict acute renal failure. Rapid Review Rhabdomyolysis RFs: trauma, heat, alcohol/drugs, exercise CPK > 5 times ULN Urine: tea-colored, + for blood but - for RBCs, myoglobinuria Hyperkalemia, hyperphosphatemia, hypocalcemia Immediate ECG Rx: IVF, bicarbonate Complications: DIC, ARF, compartment syndrome

Question: What organisms cause SBI in neonates?

Answer: Organisms responsible for SBI in neonates are those found in the vaginal canal: Group B Streptococcus, Escherichia coli, and Listeria monocytogenes. Rapid Review Crying/Fussiness Rule out trauma, abuse, infection Discharge if easily consoled, no identifiable cause

Question: A 1:1 mixture of water & vinegar is beneficial for both the treatment and the prevention of what?

Answer: Otomycosis.

Question: What is Hutchinson sign?

Answer: Periungual pigmentation.

Question: What is the most common type of delusion?

Answer: Persecutory. Rapid Review Delusions Fixed, false belief Persecutory most common Schizophrenia, bipolar

Question: What is the treatment for familial adenomatous polypsosis (FAP)?

Answer: Prophylactic colectomy.

Question: A yellow-green fluorescence seen during Wood's lamp examination can detect early infection of which bacterium?

Answer: Pseudomonas, common in contact lens wearers. Rapid Review Acute Angle-Closure Glaucoma Precipitated by pupillary dilation Acute unilateral painful vision loss Cloudy cornea, perilimbic flush, fixed mid-dilated pupil Nausea/vomiting ↑ IOP (>21 mmHg) Topical ßBs, carbonic anhydrase inhibitors, steroids, miotics

Question: What labs should be considered in pityriasis rosea?

Answer: RPR or VDRL to rule out syphilis. Rapid Review Pityriasis Rosea Patient with a history of a larger lesion 1 week prior, ​"Herald Patch" Complaining of rash on the back PE will show diffuse papulosquamous rash on the trunk, "Christmas tree-like" distribution Treatment is self-limiting disease, itching with antihistamines

Question: What is the most common cause of acute adrenal insufficiency in the United States?

Answer: Rapid withdrawal of exogenous steroids in patients on long-term therapy. Rapid Review Corticoadrenal Insufficiency 1°: Addison disease Weakness, fatigue, anorexia, weight loss Hyperpigmentation (1° disease) Hyponatremia and hyperkalemia (1° disease) ↑ ACTH = 1° disease ↓ ACTH = 2° disease Hydrocortisone Fludrocortisone (1° disease)

Question: What is the first-line treatment for an immunocompetent adult with Herpes zoster?

Answer: Famciclovir or Valacyclovir.

Question: What are the most common clinical manifestations of PCP?

Answer: Fever, non-productive cough, progressive dyspnea and chest discomfort. Rapid Review Pneumocystis Pneumonia (PCP) Patient with a history of HIV Complaining of gradual onset of non-productive cough Labs will show CD4 < 200, increased LDH CXR will show bilateral infiltrates (bat wing pattern) Most commonly caused by Pneumocystis jirovecii Treatment is TMP-SMX

Question: Name two medications used to treat orthostatic hypotension?

Answer: Fludrocortisone and pyridostigmine. Rapid Review Orthostatic Hypotension Decrease in systolic blood pressure ≥ 20 Decrease in diastolic blood pressure ≥ 10 Inadequate physiologic response to postural changes

Question: What formula is used to calculate the serum osmolality?

Answer: (2 x Na) + (BUN/2.8) + (glucose/18) + (ethanol/3.7).

Question: What diameter of induration constitutes a positive tuberculin skin test in children less than 4 years of age without tuberculosis risk factors?

Answer: 10 mm.

Question: What is the maximum daily recommended dose of vitamin B6?

Answer: 50 mg daily.

Question: What does the highest (worst) TIMI score of 7 indicate?

Answer: A 41% chance that in the next 14 days the patient will die, have a new or recurrent MI or have severe recurring ischemia which requires revascularization. Rapid Review Ischemic Heart Disease #1 cause of death in USA RFs: family hx, smoking, HTN, DM, cholesterol, male, age >45 Stable angina: activity → chest pain (CP), relieved by rest, NTG Unstable angina: CP at rest CP, dyspnea, diaphoresis, nausea, hiccups, radiation to shoulder/jaw/back Elderly, diabetics, females, hx of stroke or HF: ↑ risk for atypical presentation Earliest ECG sign of MI: hyperacute T waves Up to 50% of ECGs are negative or nonspecific Highest S/S: troponin I

Question: Status migrainosus is defined by a headache that lasts at least how long?

Answer: A diagnosis of status migrainosus requires an ongoing migraine of at least 72 hours in duration. Rapid Review Migraine Headache Gradual onset, unilateral > bilateral, throbbing, pulsating First episode: < 30 years old F > M Triggers: cheese, OCPs, pregnancy, menstruation Without aura: most common, N/V, photophobia, phonophobia Aura: scotoma, flashing lights, sounds Abortive rx: triptans, DHE, antiemetics, NSAIDs Ppx: ßBs. CCBs, TCAs Triptans, DHE: contraindicated in HTN or CV disease

Question: What is the most serious complication of impetigo?

Answer: Acute glomerulonephritis. Rapid Review Impetigo Patient will be a child < 6 y/o Complaining of NON painful, pruritic lesions on face PE will show "honey colored," weeping Most commonly caused by S. aureus Treatment is topical mupirocin Complications: poststreptococcal glomerulonephritis

Question: What is first line therapy for spinal stenosis?

Answer: Adequate pain management, physical therapy, exercise, and weight loss. Rapid Review Spinal Stenosis Radicular pain Pain: ↑ walking downhill, ↓ walking uphill MRI Rx: physical therapy, surgery

Question: Is estrogen replacement therapy (ERT) indicated in primary prevention of osteoporosis in women?

Answer: Although once believed to be beneficial, ERT therapy is no longer indicated for primary prevention of osteoporosis. Rapid Review Osteoporosis Decline in bone mass with aging → ↑ bone fragility + ↑ fracture risk F > M Risk factors: alcohol, steroid use, whites, Asians Height loss Most common fracture: vertebral body compression fractures DEXA scan T-score ≤ -2.5 Prevention: weight-bearing exercises, calcium, vitamin D, smoking cessation Pharmacologic rx: bisphosphonates (1st line)

Question: What is the name for an episode of transient monocular blindness caused by acute retinal ischemia?

Answer: Amaurosis fugax. Rapid Review Transient Ischemic Attack Transient episode of neurological dysfunction without acute infarction 10% of TIA patients will have a stroke within 90 days Aspirin + dipyridamole or clopidogrel monotherapy ABCD2 score: predicts likelihood of subsequent stroke within 2 days

Question: What is the average difference in resting heart rate in a pregnant woman vs. a nonpregnant woman?

Answer: Approximate increase of 10 to 15 beats/minute in pregnancy. Rapid Review Normal Pregnancy ↑: Blood volume, CO, TV ↓: FRC, SVR Respiratory alkalosis: 3rd trimester Doppler heart tone at 10 weeks HR: ↑ 10-15 bpm BP: ↓ in 2nd trimester, normalizes in 3rd Serum ß-hCG: doubles every two days in early pregnancy Fundal height: 12 weeks: pubic symphysis 20 weeks: umbilicus 20-32 weeks: height (cm) above symphysis = gestational age (weeks) Transvaginal ultrasound: IUP visualized when ß-hCG > 1500 Transabdominal ultrasound: IUP visualized when ß-hCG > 4000

Question: What other bacteria may mimic symptoms of Group A Strep pharyngitis?

Answer: Arcanobacterium haemolyticum, group C streptococcus.

Question: What is the difference between aspiration pneumonitis and aspiration pneumonia?

Answer: Aspiration pneumonitis is an inflammatory reaction which results from the aspiration of gastric acidic fluid from the stomach. Aspiration pneumonia is a bacterial pneumonia caused by aspiration. Rapid Review Bacterial Pneumonia S. pneumonia: most common, rusty colored sputum, rigors, gram+ paired lancets Klebsiella: alcoholics, currant jelly sputum, bulging fissures, S. aureus: IVDA, postinfluenza, elderly, gram+ cocci in clusters H. influenzae: COPD, gram negative pleomorphic rods Pseudomonas: cystic fibrosis, nursing home resident and cyanosis Health care associated pneumonia: pseudomonas, MRSA Outpatient, healthy: macrolide or doxycycline Outpatient, comorbidity: respiratory tract fluoroquinolone (RTF) Inpatient: RTF ICU: antipneumococcal ß-lactam (ceftriaxone or cefotaxime) + either azithromycin or an RTF

Question: What is the mechanism of thrombocytopenia in idiopathic thrombocytopenic purpura?

Answer: Autoantibodies directed against platelets lead to platelet destruction. Rapid Review Idiopathic Thrombocytopenic Purpura (ITP) Patient will be a child 2 - 6 yrs old With a history of recent viral infection Complaining of red spots on skin or easy bleeding PE will show petechiae, purpura, and gingival bleeding Labs will show platelets < 50,000 µL Most commonly caused by antiplatelet antibodies Treatment is observation, steroids, IVIG

Question: What are some common diet modifications you would teach to a patient with esophagitis?

Answer: Avoid spicy and acidic foods, avoid alcohol and tobacco, take small bites and chew thoroughly, stick with soft foods such as applesauce, pudding, and protein shakes, and drink liquids through a straw. Rapid Review Esophagitis Most common cause: GERD Infectious esophagitis: immunocompromise Infectious etiologies: Candida > HSV, CMV Odynophagia, dysphagia Oral lesions not always present Treatment: Candida: fluconazole CMV: ganciclovir HSV: acyclovir

Question: In what age group are physeal injuries most common?

Answer: Boys aged 12-15 and girls aged 9-12. Rapid Review Salter-Harris Fractures I: S (Slipped epiphysis) II: A (fracture Above physis), most common III: L (fracture beLow physis) IV: T (fracture Through physis) V: R (wRecked physis) I/II rx: nonoperative IV/V rx: surgery required Negative radiographs do not r/o a Salter I fracture

Question: How is Pneumocystis jirovecii pneumonia transmitted?

Answer: By airborne route. Rapid Review Pneumocystis Pneumonia (PCP) Patient with a history of HIV Complaining of gradual onset of non-productive cough Labs will show CD4 < 200, increased LDH CXR will show bilateral infiltrates (bat wing pattern) Most commonly caused by Pneumocystis jirovecii Treatment is TMP-SMX

Question: Which type of foot anomaly is plantar fasciitis common in?

Answer: Cavus feet.

Question: What algorithm is used to determine the likelihood of bacterial pharyngitis and the need for antibiotics?

Answer: Centor criteria Rapid Review Pharyngitis Viral > bacterial Bacterial GAS (S. Pygoenes) Centor criteria: fever, cervial lymphadenopathy, tonsillar exudate, no cough Rx: PCN, erythromycin (PCN allergic), steroids Complications: rheumatic fever, PSGN, abscess

Question: What are some triggers that can precipitate cardiac arrest in long QT syndrome?

Answer: Depending on the specific channel affected, various triggers can precipitate cardiac arrest including exercise, sudden loud noises or sleep.

Question: Due to increased antibiotic resistance, what is the current recommended treatment for gonorrhea?

Answer: Dual therapy with ceftriaxone and either azithromycin or doxycycline. Rapid Review Condyloma Acuminata Cauliflower-like coalescence of warts HPV (6, 11) HPV: most common STD

Question: What is the treatment for lead poisoning?

Answer: Edetate disodium calcium (EDTA) or succimer.

Question: Which organism is the most common cause of spontaneous bacterial peritonitis?

Answer: Escherichia coli. Rapid Review Spontaneous Bacterial Peritonitis Portal HTN → bowel edema → bacterial migration from GI tract E. Coli, Streptococcus spp Patient with chronic liver disease Fever + ascites Peritoneal fluid: PMNs > 250 mm3 Immediate IV antibiotics (third-generation cephalosporin)

Question: What GI disorder is associated with erythromycin use in neonates?

Answer: Hypertrophic pyloric stenosis.

Question: What is the initial treatment for hypercalcemia?

Answer: IV hydration.

Question: What class of bacteria is Bartonella henselae?

Answer: Intracellular gram-negative rod. Rapid Review Cat Scratch Disease Patient with a history of scratch by a kitten or cat Complaining of subacute, regional lymphadenitis PE will show a vesicle, which became an erythematous papule Most commonly caused by Bartonella henselae Treatment is mainly supportive, azithromycin if needed

Question: What would the peripheral smear show in a patient with ehrlichiosis?

Answer: Intracellular morulae.

Question: What is digoxin used for?

Answer: Its positive-inotropic and antidysrhythmic effects, at therapeutic doses, are used in treating atrial fibrillation and congestive heart failure. Rapid Review Digoxin Effect Not a marker of toxicity--only indicates patient is taking digoxin Downsloping ST depression with a characteristic slurred appearance ("Salvador Dali mustache") Flattened, inverted, or biphasic t waves Shortened QT interval

Question: What is the most common cause of bilateral Bell's palsy?

Answer: Lyme disease. Rapid Review Bell's Palsy Patient with a history of viral prodrome Complaining of waking up with unilateral facial nerve paralysis, hyperacusis and taste disturbance PE will show CN VII nerve palsy that does not spare the forehead Most commonly caused by HSV Treatment is prednisone, artificial tears, tape eyelid shut Comments: Bilateral: Lyme disease, infectious mononucleosis

Question: If there is severe tenderness and swelling at the medial malleolus, which fracture must be suspected?

Answer: Maisonneuve fracture defined by fracture of the medial malleolus, disruption of the syndesmosis and fracture of the proximal fibula. Rapid Review Ankle Sprain Patient with a history of ankle inversion PE will show pain and swelling Imaging will show partial or complete tearing of ligaments Most commonly injured anterior talofibular ligament (ATFL) Treatment is RICE therapy Comments: Ottawa Rules to determine imaging

Question: What classification of anemia does vitamin B12 deficiency result in and what is the corresponding mean corpuscular volume (MCV)?

Answer: Megaloblastic anemia, MCV >100. Rapid Review Vitamin B12 (Cobalamin) Deficiency Patient will be a vegan Complaining of fatigue, weakness and peripheral neuropathy PE will show pallor and glossitis Labs will show MCV > 100, hypersegmented neutrophils, elevated homocysteine, elevated methylmalonic acid Treatment is parenteral vitamin B12 Comments: only vitamin B12 deficiency results in neurological symptoms

Question: Other than ciprofloxacin and azithromycin, what other antibiotic is recommended for use in an adult with noninvasive traveler's diarrhea?

Answer: Rifaximin. Rapid Review Traveler's Diarrhea Patient with a history of recent travel Complaining of abrupt onset of watery diarrhea, nausea, and abdominal cramping Most commonly caused by enterotoxigenic Escherichia coli (ETEC) Treatment is rehydration and ciprofloxacin

Question: What is the most common bacterial cause of pneumonia in hospitalized patients?

Answer: S. pneumoniae. Rapid Review Bullous Myringitis S. pneumoniae Fluid filled vesicles on TM Pneumonia + otalgia Beyond The Boards Bullous myringitis is not associated with mycoplasma pneumoniae: Bullous myringitis is a presentation of acute otitis media in which bullae are seen on the tympanic membrane. Despite earlier belief that this condition was associated with mycoplasmal infection, more recent evidence indicates that the prevalence of viral, bacterial, or mycoplasmal infection is the same in bullous myringitis as in nonbullous otitis media.

Question: What is the most common cause of intermittent solid food dysphagia and food impaction?

Answer: Schatzki's ring (also known as B ring) occurs at the gastroesophageal junction at the distal margin of the lower esophageal sphincter.

Question: What is a potential neurologic complication of pertussis infection?

Answer: Seizures. Rapid Review Pertussis ("Whooping Cough") Whooping cough: expiratory cough, inspiratory whoop Child with Incomplete immunization hx URI sx → paroxysmal stage → convalescence Macrolide

Question: What is SCIWORA?

Answer: Spinal Cord Injury Without Radiographic Abnormalities. This is seen in children as a result of transient subluxation. Rapid Review Anterior Cord Syndrome PE will show loss of motor, pain, and temperature below injury Most commonly caused by flexion injury Comments: proprioception and vibration intact

Question: What theoretical complication can infusion of intravenous calcium cause in a patient taking digitalis?

Answer: Stone heart, an irreversible, non-contractile state of impaired diastolic relaxation.

Question: What is the most common cause of bacterial meningitis in adults?

Answer: Streptococcus pneumoniae.

Question: What are the cutaneous side effects of topical corticosteroid applied to normal skin?

Answer: Thinning, atrophy, telangectasias, purpura, and hypertrichosis. Rapid Review Atopic Dermatitis (Eczema) Patient with a history of asthma or hay fever Complaining of itchy, scaly rash on arms, often worse in the winter PE will show thick, leathery, hyperpigmented areas on flexor surfaces Treatment is topical corticosteroids, lubricating ointments

Question: What is the treatment for a fused labia minora?

Answer: Topical estrogen cream.

Question: How frequently should children receive fluoride varnish application in order to prevent early dental caries?

Answer: Twice-yearly application.

Question: How many grams of sodium can one eat per day on a low sodium diet?

Answer: Under 2 grams. Rapid Review Chronic Heart Failure Treatment Lifestyle modifications Diuretics: used for acute pulmonary edema, no mortality benefit ACEIs: ↓ mortality in all classes BBs: ↓ mortality in classes II, III, IV Hydralazine with nitrates: ↓ mortality in African-Americans Digoxin: used in refractory systolic dysfunction, no mortality benefit Spironolactone: ↓ mortality in class III/IV Isolated diastolic dysfunction: HR and BP control Advanced treatments: mechanical assist device, ICD, heart transplantation

Question: Which test can confirm H. pylori eradication?

Answer: Urea breath test. Rapid Review Peptic Ulcer Disease Most common cause of UGIB RFs: smoking, H. pylori, NSAIDS, ASA, steroids Duodenal > gastric Duodenal: pain decreases food Gastric: pain greater with food Proton pump inhibitors, endoscopy Complications: perforation, gastric outlet syndrome

Question: What is the preferred method of prophylactic endoscopic treatment of esophageal varices to help prevent future bleeding?

Answer: Variceal band ligation as endoscopic variceal ligation decreases the risk of first bleeding and the mortality rate. Rapid Review Esophageal Varices Caused by portal hypertension Chronic liver disease, alcoholism Associated with massive UGIB Hemodynamic support Octreotide, vasopressin, Sengstaken-Blakemore tube

Question: What is the most common long term complication of idiopathic intracranial hypertension?

Answer: Vision loss. Rapid Review Idiopathic Intracranial Hypertension (Pseudotumor Cerebri) Young obese females Vitamin A toxicity, steroids, tetracycline ↓ CSF absorption HA + visual sx Papilledema, CN VI palsy ↑ opening pressure on LP Acetazolamide, serial LPs Weight loss

Question: What are some methods to decrease pain associated with injection of lidocaine?

Answer: Warming the solution, buffering the solution with sodium bicarbonate, and injecting into the wound rather than through intact skin. Rapid Review Injectable lidocaine for anesthesia: Maximum dose lidocaine is 5 mg/kg Maximum dose lidocaine with epinephrine is 7 mg/kg 1% lidocaine solution has 10 mg/mL; 2% solution has 20 mg/mL. Use LET gel on open wounds EMLA is used for intact skin Signs of lidocaine toxicity include twitching, tremors, convulsions, bradycardia, hypotension, and cardiac arrest

Question: In intrahepatic cholestasis of pregnancy do the number of adverse fetal outcomes increase as serum bile acid levels rise?

Answer: Yes.

Which of the following best defines delusions? Erroneous beliefs that usually involve a misinterpretation of perceptions or experiences Hallucinations occurring in the absence of insight into their pathological nature Marked disorientation, confusion, and fluctuating consciousness Sensory perceptions without external stimulation

Correct Answer ( A ) Explanation: Delusions are erroneous beliefs that usually involve a misinterpretation of perceptions or experiences. Their content may include a variety of themes (persecutory, referential, somatic, religious, or grandiose). In persecutory delusions the person believes he or she is being tormented, followed, tricked, spied on, or ridiculed. In referential delusions the person believes that certain gestures, comments, passages from books, newspapers, song lyrics, or other environmental cues are specifically directed at him or her. Psychosis (B) is restricted to delusions or prominent hallucinations, with the hallucinations occurring in the absence of insight into their pathological nature. Hallucinations (D) are sensory perceptions without external stimulation. Hallucinations may occur in any sensory modality (auditory, visual, olfactory, gustatory, and tactile). Auditory hallucinations are the most common. Delirium (C) is characterized by marked disorientation, confusion, and fluctuating consciousness.

An elderly man with congestive heart failure presents to the ED with a complaint of "not feeling normal." He is on several different medications. His ECG is shown above. Which of the following medications is the most likely cause of this ECG abnormality? Digoxin Furosemide Metoprolol Warfarin

Correct Answer ( A ) Explanation: Digoxin is a cardiac glycoside derived from the foxglove plant. It is used to increase the force of myocardial contraction in systolic heart failure and to decrease AV nodal conduction in atrial fibrillation. It works by inactivating the Na+ K+ ATPase pump on the cardiac cell membrane which leads to increased intracellular calcium and extracellular potassium. In individuals taking digoxin, there is a characteristic ECG pattern commonly referred to as the "digoxin effect." The presence of the "digoxin effect" on the ECG is not a marker of toxicity, rather it indicates that the patient is taking digoxin. The "digoxin effect" is characterized by (1) downsloping ST depression with a characteristic "slurred" appearance; (2) flattened, inverted, or biphasic T waves; (3) shortened QT interval. The overall morphology is commonly compared to the shape of a mustache. Furosemide (B) is a diuretic that is not directly related to changes in ECG morphology. However, in extreme cases it can lead to electrolyte abnormalities that are represented by ECG changes. Metoprolol (C) and other beta-blockers may cause a first-degree heart block (PR interval > 200 msec). Warfarin (D) does not affect the cardiac electrical pathway to produce ECG changes.

An elderly woman with a recent 21-day course of oral antibiotics for pneumonia presents with difficulty swallowing. Her past medical history is significant for hypertension, interstitial cystitis and alcoholism. A barium swallow study shows longitudinal plaques along the lining of the esophagus. Which of the following is the most likely causative agent for this patient's symptoms? Candida Cytomegalovirus Herpes simplex virus type I Mycobacterium tuberculosis

Correct Answer ( A ) Explanation: Esophagitis refers to inflammation of the esophagus. One etiologic category is infectious, with Candida being the most common agent. Risk factors for candidal esophagitis includes long-term antibiotic use, cancer, radiotherapy, chemotherapy, advanced age, malnutrition and alcoholism. Esophagitis presents with dysphagia, or difficulty swallowing. Other complaints include "food getting stuck", drooling, nasal regurgitation, nausea, vomiting, dysarthria and cough. Diagnostic procedures include upper endoscopy with biopsy, in which an endoscope is passed into the esophagus, or an upper GI series (also known as a barium swallow, or esophagography), in which radiographs are taken after ingesting barium contrast. For infectious esophagitis specifically, a double-contrast esophagography better detects the classic 'shaggy' appearance of esophageal longitudinal plaques and psuedomembranes. Treatment includes antifungals, antivirals, pain medication preparations, acid reducers and diet modification. Cytomegalovirus (B) esophagitis almost never occurs in immunocompetent patients. The vast majority of these patients are infected with the HIV virus. Herpes simplex virus type I (C) is the second most common cause of infectious esophagitis, after Candida. It is common in immunocompromised patients with AIDS, an underlying malignancy, or a debilitating illness. Tuberculosis (D) esophagitis, although rare, occurs primarily in patients with advanced pulmonary or mediastinal tuberculosis.

You examine an eight-year-old girl in the pediatric intensive care unit who is complaining of abdominal pain. She has been in the unit for six days for close monitoring and pain control after her spinal surgery. She has been immobilized since the day of surgery and now complains of nausea, constipation, and abdominal pain. Her ECG shows a shortened QT interval. Which of the following electrolyte abnormalities can explain her findings? Hypercalcemia Hypernatremia Hypokalemia Hypomagnesemia

Correct Answer ( A ) Explanation: Hypercalcemia results when the entry of calcium into the circulation exceeds the excretion of calcium into the urine or deposition in bone. This occurs when there is accelerated bone resorption, excessive gastrointestinal absorption, or decreased renal excretion of calcium. Immobilization can cause hypercalcemia due to increased bone resorption. Patients with mild hypercalcemia (calcium < 12 mg/dL) may be asymptomatic, or they may have nonspecific symptoms such as constipation, fatigue, and depression. Serum calcium of 12 to 14 mg/dL may be well-tolerated chronically. On the other hand, an acute rise may cause marked symptoms such as polyuria, polydipsia, dehydration, anorexia, nausea, muscle weakness, and changes in sensorium. Acute hypercalcemia also shortens the myocardial action potential, which is seen in a shortened QT interval. Treatment for hypercalcemia is aimed both at lowering the serum calcium concentration and in treating the underlying disease, if possible. Hypernatremia (B) happening acutely results in the rapid decrease in brain volume that can lead to rupture of the cerebral veins while chronic hypernatremia results in fewer neurologic symptoms. Hypokalemia (C) results in muscle cramps, rhabdomyolysis, and myoglobinuria. It also produces characteristic changes on the ECG such as depression of the ST segment, decrease in amplitude of the T wave, prolonged QT interval and increase in amplitude of U waves. Hypomagnesemia (D) manifests with neuromuscular hyperexcitability such as tetany, seizures, and involuntary movements. It can also produce ECG changes such as widening of the QRS complex, peaking of T waves, prolongation of the PR interval, progressive widening of the QRS complex, and diminution of the T wave.

Which of the following treatments is recommended as first-line management for patients with idiopathic intracranial hypertension? Acetazolamide Corticosteroids Serial lumbar punctures Ventricular Shunt

Correct Answer ( A ) Explanation: Idiopathic intracranial hypertension is caused by increased brain water content and decreased cerebrospinal fluid (CSF) outflow. Patients typically present with generalized headache that is gradual in onset. The headache may be worsened by eye movements. Additionally, any valsalva maneuver decreases cerebral venous return and will worsen the headache. Patients will often have visual complaints including transient vision loss secondary to ischemia of the visual pathways. Other typical symptoms include nausea, vomiting and lightheadedness. Fundoscopic examination reveals papilledema. Diagnosis is made based on an increased opening pressure found on lumbar puncture. Treatment focuses on decreasing CSF production initially with acetazolamide. Acetazolamide is a carbonic anhydrase inhibitor that can be used alone or in combination with furosemide. Corticosteroids (B) have shown some benefit but the mechanism of action is unknown. Serial lumbar punctures (C) to remove excess CSF is effective but invasive and painful. Ventricular shunt placement (D) and optic nerve sheath fenestration represent the last lines of therapy.

A 10-year-old boy has recently moved to the United States from India with his mother. He previously received the Bacille Calmette-Guerin (BCG) immunization. He denies cough, fever, or weight loss. A tuberculin skin test is placed. 48 hours later, the area of induration at the injection site measures 10 mm in diameter. Which of the following is the most appropriate next step in management? Obtain a chest radiograph Place the patient in airborne isolation Reassure the family Repeat the skin test in six months

Correct Answer ( A ) Explanation: Injection-site induration of a tuberculin skin test (TST) is considered positive at greater than or equal to 10 mm diameter of induration in a patient who was born in a region with a high prevalence of tuberculosis. Many patients from high-prevalence regions have received the BCG vaccine prior to arrival in the United States. Receipt of the BCG does not impact the threshold of positivity for the TST. Whether a patient who has received the BCG vaccine will have a positive TST is extremely variable and depends on a number of factors, including age at vaccination, number of doses received, time lapse since immunization, the strain contained in the vaccine, the number of prior TST's, and patient's immune and nutritional status. Thus, it is any patient with a positive tuberculin skin test requires a chest radiograph regardless of vaccination status. Radiographic evidence of granulomas, calcifications, or adenopathy are consistent with tuberculosis and cannot be caused by the BCG vaccine. Patient's with active tuberculosis should be immediately placed in airborne isolation (B). However, as the patient has a positive tuberculin skin test but no evidence of tuberculosis disease, isolation is not yet required. However, if his chest radiograph is positive, he will require immediate airborne isolation. Reassuring the family (C) is not appropriate, as the patient's 10 millimeters of induration is considered positive in any patient born in a high-prevalence region, even if the patient has received the BCG vaccine in the past. Repeating the skin test in six months (D) is likely to be unhelpful, as the test would still be expected to be positive. Instead, active pulmonary tuberculosis must be excluded through review of a chest radiograph.

A 10-year-old girl presents for a well-child check. On exam, she is noted to have heavy plaque accumulations on her teeth as well as edema and erythema of the gingival tissues. There is no evidence of gingival attachment loss or recession, tooth mobility, or tooth loss. Which of the following best characterizes the patient's condition? Hormonal changes may predispose its development It is often associated with irreversible structural loss The incidence and severity decrease from childhood to adolescence Treatment may include debridement, scaling, and root planing

Correct Answer ( A ) Explanation: Periodontal disease (broadly classified as either gingivitis or periodontitis) is prevalent in children and adolescents, affecting nearly every individual to some degree by puberty. Chronic gingivitis is the most common form of periodontal disease in children. It is characterized by inflammation of the gingival tissues in the absence of loss of bone or connective tissue attachment. The most common cause of gingivitis is poor oral hygiene; however several local factors may also play a role in the development of the condition. Hormonal changes associated with puberty and pregnancy often cause inflammation and hypertrophy of the gingiva. Other local factors include chronic mouth breathing, orthodontic appliances or braces, teeth crowding, and tooth eruption. Systemic risk factors include drugs, such as anticonvulsants, immunosuppressants, and calcium-channel blockers, immunocompromised states (e.g. HIV, leukemia), and vitamin C deficiency. Tobacco use is also a major risk factor for the development of periodontal disease. Gingivitis can progress to periodontitis, which is characterized by gingival inflammation with loss of supporting tissue structures including alveolar bone. In contrast to periodontitis, gingivitis is not associated with irreversible structural loss (B). Even longstanding gingivitis typically resolves over time after effective oral hygiene measures are adopted. Periodontal disease is uncommon in very young children, however the incidence and severity increases from childhood to adolescence (C), peaking at puberty. Treatment for simple gingivitis includes instituting appropriate oral hygiene. Periodontitis may require more aggressive treatment, such as debridement, scaling, and root planing (D).

A 48-year-old man with a history of HIV presents to your office with complaints of fever, cough and shortness of breath. Laboratory testing reveals a CD4 count of 130 cells/microL. Which of the following findings is most likely to be seen on chest X-ray? Ground glass opacification Hampton's hump Honeycombing Kerley B lines

Correct Answer ( A ) Explanation: Pneumocystis jirovecii pneumonia (PCP), previously called Pneumocystis carinii pneumonia, is an opportunistic infection found in individuals infected with HIV who have CD4 counts of less than 200 cells/microL. Symptoms of PCP develop gradually, with fever, cough and shortness of breath being the most common complaints. Patients may also present with fatigue, weight loss, chest pain and chills. Chest X-ray will reveal bilateral, fine, diffuse opacification, often with a ground glass appearance. Patients with HIV whose CD4 counts drop below 200 cells/microL should be given prophylaxis with trimethoprim-sulfamethoxazole and treatment of the infection is with the same agent. Kerley B lines (D) are caused by a thickening of the subpleural interstitium and are often seen in patients with pulmonary edema. Hampton's hump (B) is a wedge of airspace opacity on the periphery of the lung and is seen with pulmonary embolism. Honeycombing (C) is found with end-stage interstitial lung disease such as pneumoconiosis.

Which of the following is an indication for prophylaxis against Pneumocystis jirovecii pneumonia in individuals infected with HIV? CD4 count of 125 CD4 count of 1500 Individuals taking anti-retroviral therapy Pregnant women with HIV regardless of CD4 count

Correct Answer ( A ) Explanation: Pneumocystis jirovecii pneumonia (PCP), previously called Pneumocystis carinii pneumonia, is the most common cause of death in individuals with AIDS-related complications. Primary prophylaxis is indicated when CD4 counts fall below 200. Risk factors for the development of this opportunistic infection include a history of PCP, decreased CD4 count, as well as undiagnosed weight loss, oropharyngeal candidiasis, night sweats and fever in individuals with CD4 counts above 200 cells. Definitive diagnosis is by cytopathologic or histopathologic evidence of the organism in induced sputum samples, bronchoalveolar lavage fluid, or tissue. Trimethoprim-sulfamethoxazole is the recommended medication for prophylaxis and is also used for treatment of the disease. Some experts recommend starting prophylaxis against PCP when the CD4 counts are between 200-250 cells if frequent monitoring of CD4 count is not possible, otherwise prophylaxis is not recommended for CD4 counts above 200 cells (B). Indication for PCP prophylaxis is the same, regardless of whether the patient is on anti-retroviral therapy (C) or not. Pregnant women with HIV (D) follow the same guidelines for PCP prophylaxis as non-pregnant individuals.

Where is the uterine fundus palpable at 36 weeks gestation? At the umbilicus At the xiphoid process Between the pubic symphysis and umbilicus Between the umbilicus and xiphoid process

Correct Answer ( B ) Explanation: At six to eight weeks, the gravid uterus is approximately the size of an orange. At 12 weeks, the top of the fundus should be at the level of the symphysis pubis; at 20 weeks, at the level of the umbilicus; and at 36 weeks, at the level of the xiphoid process. Subsequently, the fetus descends into the pelvis, and the fundal height may decrease. The fundus is at the umbilicus (A) at 20 weeks; between the pubic symphysis and umbilicus (C) at 16 weeks; and between the umbilicus and xiphoid process (D) at 28 weeks.

A 3-year-old girl is brought in after her mother noticed a rash and bruising over her trunk and extremities. She also has intermittent epistaxis over the past few days. She had an upper respiratory illness 2 weeks ago but otherwise is well. Examination reveals a well-appearing child with scattered petechiae. Hemoglobin is 12 g/dL, WBC 8,000, INR 1.0, and platelets 8,000. Which of the following is the most appropriate initial treatment? Corticosteroids and intravenous immunoglobulin Observation Platelet transfusion Splenectomy

Correct Answer ( A ) Explanation: The child has idiopathic thrombocytopenic purpura (ITP), an acquired autoimmune disorder that results in platelet destruction. It often follows a viral illness and is more common in children than adults. It is characterized by thrombocytopenia in the absence of other bone marrow pathology. Signs and symptoms include petechiae, purpura, and gingival bleeding. Management depends on degree of thrombocytopenia and presence of bleeding, and should be performed in consultation with a hematologist. In general, patients with platelet count of 10,000-20,000 µL and mucosal bleeding or those with platelet counts < 10,000 µL and no bleeding are treated with corticosteroids or intravenous immunoglobulin (IVIG) or both. Asymptomatic patients with platelets > 20,000µL can be observed, as the condition is often self-limited. Observation (B) is indicated for asymptomatic patients with platelets > 20,000/µL. Platelet transfusion (C) is only indicated for life-threatening bleeding after administration of IVIG and steroids. Splenectomy (D) is rarely required for ITP and is only considered in refractory cases.

Which of the following is the recommended treatment of benign esophageal stricture disease? Esophageal dilation Esophogram H2 blockers Radiation treatment

Correct Answer ( A ) Explanation: The foundation for treatment of benign stricture disease is esophageal dilation. Intrinsic strictures are most common, with acid or peptic causes accounting for the majority of cases (60-70%). The different types of dilators used in treatment are (1) mercury-filled, rubber Maloney bougies, (2) wire-guided rigid Savary-Gilliard dilators, and (3) balloon dilators, which can either be through-the-scope (TTS) or wire-guided. Choice of dilator often depends on anatomy of the stricture and operator experience. In general, Maloney bougies are used in uncomplicated short straight strictures. Complications of esophageal dilation are uncommon and include perforation, bleeding, and bacteremia. Patients with radiation-induced or malignant strictures are at higher risk of perforation. To minimize the risk of perforation, the "rule-of-threes" applies—that is, no more than three sequential dilators to be used per session. The goal of esophageal dilation is to obtain an objective diameter greater than 15 mm. Approximately 90% of patients in whom the esophagus was dilated to 15 mm have no recurrence at 24 months. Esophogram (B) is a test used to help diagnose esophageal stricture and is not used in treatment. Proton pump inhibitors (PPIs) are more effective in the prevention of stricture recurrence than H2 blockers (C). Finally, radiation treatment (D) can lead to the formation of a stricture and is not used in treatment.

A 4-month-old infant is brought to clinic with concerns of rash. Over the past week, his mother noted a red rash over both of his cheeks. The rash has neither spread nor remitted. He has otherwise been healthy. On examination, he exhibits erythematous, scaling, excoriated plaques over both cheeks. What is the most likely diagnosis? Atopic dermatitis Impetigo Irritant contact dermatitis Seborrheic dermatitis

Correct Answer ( A ) Explanation: The infant has an eczematous rash over both cheeks. Both the description of his rash and its location are consistent with atopic dermatitis. Atopic dermatitis is an inflammatory skin condition that affects children and adults of all ages. It is a result of an overactive response to a variety of stimuli, including food allergens, inhaled allergens, irritant cosmetics, weather and humidity changes, pollution, and stress. Atopic dermatitis commonly causes diffuse xerosis, as well as eczematous plaques, papules, and crusting. In infants, plaques are most commonly distributed over the cheeks, as well as the extensor surfaces of the extremities. In older children, the face is less often involved, and the flexor surfaces of the extremities, such as the antecubital fossa, are frequently affected. In addition to causing pruritis and irritation, the compromised skin integrity in atopic dermatitis may result in bacterial and viral superinfections. Children with eczema are also predisposed to other atopic diseases such as asthma and allergic rhinitis, as well as to food allergies. Impetigo (B) is a superficial cutaneous infection resulting from infection with S.aureus or beta-hemolytic Strep. Non-bullous impetigo is significantly more common than bullous impetigo. It begins as a papular rash, which progresses to vesicles that rupture, forming, honey-crusted plaques. Systemic symptoms of infection are typically absent. Bullous impetigo has a similar initial presentation of papules and vesicles but then progresses to bullous enlargement of vesicles. Although atopic dermatitis may become superinfected with Staph or Strep species, this child does not exhibit the typical honey crust of impetigo. Irritant contact dermatitis (C), also known as allergic contact dermatitis, occurs as a result of cutaneous hypersensitivity to environmental allergens. The rash is distributed over areas that contact the offending agent. It would be rare for irritant contact dermatitis to solely involve the bilateral cheeks as in the child above. Instead, the entire face would be involved if the area above the shirt collar were exposed to an irritant. The hands are also commonly affected by irritant contact dermatitis. Seborrheic dermatitis (D), also known as Cradle Cap, is a common pediatric skin condition that results in superficial scaling. The scales are classically greasy and yellow. Seborrheic dermatitis is distributed in areas where seborrheic glands are most concentrated such as the scalp, central face ("T zone"), preauricular skin, and intertrigenous area.

A two-year-old boy was brought to the emergency department because of vomiting. About 30 minutes prior, he was found by his parents with an open bottle containing ferrous fumarate tablets. The mother estimates about five tablets are missing, and each tablet contains 65 mg of elemental iron. The boy had three episodes of non-bloody vomiting. The physical examination is essentially normal except for tachycardia. Which of the following is the next best step? Get serum iron level four hours after ingestion Give activated charcoal immediately Obtain urinalysis Perform gastric lavage

Correct Answer ( A ) Explanation: The patient has symptoms and a history compatible with iron toxicity. The severity of an exposure is related to the amount of elemental iron ingested. Ferrous sulfate contains 20 percent elemental iron, ferrous gluconate has 12 percent, and ferrous fumarate contains 33 percent. Iron is directly corrosive to the GI mucosa, which can lead to hematemesis, melena, ulceration, infarction, and potential perforation. For significant ingestions (> 20 mg/kg of elemental iron), especially when tablets are identified on the abdominal radiograph, whole-bowel irrigation with a polyethylene glycol electrolyte lavage solution (PEG-ELS) is routinely recommended. Iron toxicity is described in four stages. The initial stage, 30 minutes to six hours after ingestion consists of profuse vomiting and diarrhea, abdominal pain, and significant volume losses, leading to potential hypovolemic shock. The second stage, six to 24 hours after ingestion, is the quiescent phase where GI symptoms typically resolve. In the third stage, occurring 12 to 24 hours after ingestion, patients develop multi-system organ failure, shock, hepatic and cardiac dysfunction, acute lung injury, and profound metabolic acidosis. Symptomatic patients and patients with a large exposure by history should have serum iron levels drawn four to six hours after ingestion. Serum iron concentrations of < 500 µg/dL four to eight hours after ingestion suggest a low risk of significant toxicity, whereas concentrations of > 500 µg/dL indicate significant toxicity. Giving activated charcoal immediately (B) is not effective because charcoal does not adsorb iron and therefore is not the decontamination of choice for iron poisoning. Gastric lavage (D) is not recommended in iron toxicity because it does not effectively remove large numbers of pills. Obtaining a urinalysis (C) would not help in the management of iron poisoning.

A 16-year-old man presents with a rash to the back for 1 week. He states that the rash started as a single patch and then spread to the rest of his back. The rash is itchy but otherwise, the patient is asymptomatic. What management is indicated? Antihistamines Cephalexin Oral corticosteroids Topical antifungals

Correct Answer ( A ) Explanation: This patient presents with pityriasis rosea and should be treated symptomatically with antihistamines as the condition is self-limiting. Pityriasis rosea presents as scaly, salmon colored, oval papules or plaques 1 - 2 cm in diameter on the trunk and proximal extremities. It usually presents in children and young adults. Classically, the diffuse rash is preceded by a herald patch 1 week prior. This lesion is larger (2-5 cm in diameter) than the others that form. Patients may also initially have fever, malaise or lymphadenopathy prior to the appearance of a rash but this is rare. The diffuse form of the rash has a Christmas tree-like distribution following the cleavage lines of the skin. Pityriasis rosea is a self-limiting disease but may take 8-12 weeks to completely resolve. The causative agent is unknown although a virus is suspected (HHV 7). There is no specific treatment for the disease and so care should be directed at relieving symptoms, most commonly itching, with antihistamines. Cephalexin (B) is a first generation cephalosporin with activity against many streptococcus and staphylococcus species, which are not implicated in pityriasis rosea. Oral corticosteroids (C) have not been shown to reduce symptoms or duration of pityriasis rosea. Although tinea infections are on the differential diagnosis for pityriasis rosea, a fungal etiology has not been shown to be causative of the disease and topical antifungals (D) do not play a role in treatment.

A 29-year-old pregnant patient presents with a three day history of diarrhea following a trip to Mexico. The patient describes having seven to eight loose, watery stools per day. She does not note any blood in the diarrhea and is otherwise asymptomatic. Vital signs are BP 116/72, HR 86, RR 15, T 38.7°C. Physical exam reveals mild dehydration, but is otherwise normal. After providing fluid replacement, what is the indicated treatment? Azithromycin Ciprofloxacin Symptomatic therapy with loperamide Trimethoprim-Sulfamethoxazole

Correct Answer ( A ) Explanation: Traveler's diarrhea is the most common illness affecting travelers to low-income parts of the world. Common causes include Enterobacteriaceae, such as E. coli, Shigella spp, and Campylobacter, as well as norovirus and rotavirus. Parasitic infection, such as Giardia or Cryptosporidium, should also be considered, especially in the event of persistent (>14 d) or refractory diarrhea. The syndrome begins abruptly with several loose or watery stools per day and may include symptoms such as nausea, vomiting, and abdominal cramping. When patients present with gross blood mixed with stools or fever, the diarrhea has evolved into dysentery, indicating invasion of the intestinal mucosa. In the situation of diarrhea with fever or bloody stools, a stool culture should be performed. Fluid replacement is always the primary treatment of traveler's diarrhea. Most cases are self-limited and resolve on their own within three to five days of treatment with fluid replacement only. Antimicrobial therapy shortens the disease duration to about one day. Antibiotics are warranted to treat diarrhea in those who develop severe diarrhea, characterized by more than four unformed stools daily, fever, or blood, pus, or mucus in the stool. In addition, some travelers desire antibiotic treatment for milder disease if the illness is a large burden on a business trip or vacation. Azithromycin is the preferred antibiotic therapy in pregnant women and children. It is also the first choice for use in South and Southeast Asia where Campylobacter is common and has high resistance to fluoroquinolones. Ciprofloxacin (B) is the antibiotic of choice for most destinations and is effective against most mucosally invasive pathogens, but is contraindicated in children and pregnancy. Antimotility agents such as loperamide (C) can provide symptomatic relief, but should not be used without antibiotics in patients with fever or bloody stools. Trimethoprim-Sulfamethoxazole (D) is no longer a recommended treatment for traveler's diarrhea due to widespread antibiotic resistance.

A mother brings her 4-year-old previously healthy daughter to clinic for vaginal itching and erythema. The mother is the primary care provider for the child and does not suspect abuse. On exam there is no discharge, the vulva appears erythematous, and the hymen is intact, thin and pinkish-red. Which of the following is the most appropriate management? Advise the avoidance of strong detergents and bubble baths Immediately call Child Protective Services Obtain gonorrhea and chlamydia DNA probes Prescribe acyclovir

Correct Answer ( A ) Explanation: Vulvovaginitis presents with vaginal irritation, pain, and pruritus. Irritant vulvovaginitis is the most common cause of genital complaints in prepubertal girls. Vulvovaginitis can be caused by sexual and non-sexually transmitted causes. Given the age of the patient in this clinical scenario and the lack of concern for sexual abuse, her vulvovaginitis is most likely due to a non-specific cause. Poor or excessive hygiene and chemical irritants such as bubble bath are the most common causes. Tight clothing is also a cause. Vulvitis refers to the external genital pruritus, burning, redness, or rash. Vaginitis implies inflammation of the vagina, which manifests as discharge with or without an odor or bleeding. Vaginitis can lead to vulvitis. Other cases of non-sexually transmitted vulvovaginitis include pinworms (Enterobius vermicularis), group A beta hemolytic Strep, Staph. Spp. and candida. There is no evidence of sexual abuse in this patient and Child Protective Services (B) does not need to be notified. Gonorrhea, Chlamydia, and herpes simplex virus are all causes of infections vulvovaginitis. However, this is unlikely in this patient. Therefore, gonorrhea and chlamydia DNA probes (C) nor acyclovir (D) is necessary.

An 8-year-old girl presents with fever, malaise and headache. On exam, she has left-sided tender axillary lymph nodes and the lesion seen above on her wrist. Which of the following is the most likely diagnosis? Brucellosis Cat scratch disease Toxoplasmosis Tularemia

Correct Answer ( B ) Explanation: Cat scratch disease is caused by Bartonella henselae which is transmitted by cat scratches (most often by kittens). In most cases, patients first present with an erythematous crusty lesion at the site of the inoculation 3 to 12 days after a scratch. One to three weeks after this primary lesion, regional lymphadenopathy appears, most commonly in the axillary, cervical and inguinal nodes and is often painful or suppurative. Patients may also develop fever, malaise, headache, and anorexia. Rare complications include bacillary angiomatosis (mainly in immunocompromised individuals), hepatosplenomegaly, encephalopathy, pneumonitis, myelitis, joint pain, parinaud oculoglandular syndrome (unilateral conjunctivitis with per-auricular lymphadenitis), neuroretinitis, and abdominal pain. Bartonella is difficult to culture and so serologic testing is the preferred method. Polymerase chain reaction may also be used but is less sensitive. Management is mainly supportive with antipyretics and analgesics. Antibiotics are indicated in immunocompromised patients or severe cases. First line drugs include trimethoprim-sulfamethoxazole, ciprofloxacin and azithromycin. Brucellosis (A) is caused by the ingestion of unpasteurized milk or undercooked meat contaminated with the gram negative bacteria Brucella. Symptoms include joint and muscle pain, profuse sweating, and fever. Toxoplasmosis (C) is caused by Toxoplasma gondii, an intracellular parasite, acquired from the ingestion of cysts in undercooked meat, through gardening, or from cat feces. Symptoms can be flu-like or involve cysts in nervous and muscle tissue, prurigo-like nodules, ecchymoses, urticaria and maculopapular lesions. Tularemia (D) is caused by Francisella tularensis which is transmitted by ticks and deer flies on rabbits and rodents. Symptoms include skin ulcers, lymphadenitis, headache, fever, fatigue, eye symptoms, vomiting, pharyngitis, hepatosplenomegaly and cough.

A 16-year-old boy presents to the office with 10 headaches over the last 3 months. These headaches last 8-10 hours, are unilateral, pulsatile, get worse with physical activity, and often accompanied by nausea with emesis. The headaches have caused him to miss school. Ibuprofen has minimal relief. Which of the following is the the most likely diagnosis? Cluster headache Migraine headache Rebound headache Tension headache

Correct Answer ( B ) Explanation: Migraine headaches are divided into two categories: those without aura (formerly known as common migraine), and those with aura (also known as classic migraine). Migraines typically present among patients 10-30 years of age. Migraine without aura is described as a recurrent headache lasting 4-72 hours. Diagnosis of migraine without aura requires two of the following characteristics: unilateral distribution, pulsatile quality, severity limited daily activities, and exacerbation by physical activity. One of the following characteristics must also be present: nausea or vomiting, photosensitivity, and sensitivity to noise or smell. Migraine with aura presents with symptoms similar to those of migraine without aura, but the headache is preceded by an aura. An aura is a reversible symptom indicative of focal cerebral dysfunction. Examples of auras include: gradual onset and spread of scotomas, scintillations, hemianoptic field defects, unilateral paresthesias or numbness, unilateral weakness, and speech disturbances. The patient in the clinical scenario meets criteria for migraine without aura. Cluster headaches (A) most often occur in middle-aged men and are often described as a recurrent, unilateral, excruciating periorbital headache that lasts from 15 minutes to 3 hours. They are associated with ipsilateral conjunctival injection, lacrimation, and nasal discharge. Rebound headaches (C) are often described as a chronic or nearly daily headache associated with frequent use of medication for acute head pain. Tension headaches (D) are the most common type of recurring headache other than migraine. Tension headaches classically present in a bandlike distribution and are exacerbated by stress, fatigue, glare, or noise.

A previously healthy 9-year-old boy presents with a sore throat. He denies coryza and diarrhea. He has a fever of 102.3° F and tender cervical lymphadenopathy. Posterior pharynx inspection reveals white exudates. Which of the following is the most likely cause of this patient's symptoms? Candida albicans Group A beta-hemolytic streptococcus Mycoplasma pneumoniae Rhinovirus

Correct Answer ( B ) Explanation: Most cases of pharyngitis are viral. However, the most common bacterial cause, Group A beta-hemolytic streptococcus (GABHS), is the cause of up to a third of childhood and tenth of adulthood cases. Transmission is via respiratory droplets, and incubation is 2-3 days. Symptoms include temperature greater than 100.4°F, tonsillar exudates, cervical adenopathy, and lack of cough. Diagnostic tools include throat culture and rapid antigen detection. Other than supportive measures, antibiotic treatment depends on several factors which determine the likelihood of bacterial infection. Penicillin, amoxicillin and first-generation cephalosporins are the drugs of choice. For penicillin allergic patients, azithromycin or clindamycin are alternatives. Candida albicans (A) can cause pharyngitis and pharyngeal exudates, however, it is almost always associated with an immunocompromised state, such as patients on chronic corticosteroids, or those with organ transplantation, or HIV infection. This patient has no such immunosuppression. Mycoplasma pneumoniae (C) is a very rare cause of bacterial pharyngitis. In addition to sore throat, the presence of cough, coryza and diarrhea are more indicative of a viral, not bacterial, pharyngitis. Rhinovirus (D) and adenovirus are common causes of viral pharyngitis.

A diagnosis of orthostatic hypotension can be made in which of the following scenarios? A drop in diastolic blood pressure of 5 mm Hg upon standing A drop in systolic blood pressure of 35 mm Hg upon standing An increase in heart rate of 35 beats per minute upon standing An increase in heart rate of 5 beats per minute upon standing

Correct Answer ( B ) Explanation: Orthostatic hypotension is defined as a decrease in systolic BP of 20 mm Hg or diastolic BP of 10 mm Hg within 3 minutes of standing from a seated or supine position. Symptoms may include positional syncope, dizziness, generalized weakness or fatigue. Once underlying cardiac, endocrine, neurologic and medication side effect etiologies are ruled out, treatment options include increasing water intake, sodium supplementation, abdominal or lower extremity binders, a physical therapy conditioning program and medications. A diagnosis of orthostatic hypotension requires a drop of 10 or more mm Hg (A) in diastolic BP upon standing. Although commonly checked along with blood pressure measurements during supine and standing position, baseline heart rate, or changes in heart rate (C), are not included in the latest definition of orthostatic hypotension. Postural orthostatic tachycardia syndrome, not orthostatic hypotension, should be considered if the heart rate increases ≥ 30 beats per minute (D) upon standing.

A 6-year old girl presents with one day of fever, mild sore throat, abdominal pain, and two episodes of non-bloody, non-bilious emesis. Abdominal exam is significant only for mild epigastric tenderness, but exudative pharyngitis and bilateral anterior cervical lymphadenopathy are noted. What is the most likely diagnosis? Food poisoning Group A Strep pharyngitis Viral gastritis Viral upper respiratory infection

Correct Answer ( B ) Explanation: Patients with Group A Strep pharyngitis may present with primary complaints of vomiting, abdominal pain, and fever with mild or no complaints of sore throat. A physical examination is essential to making the correct diagnosis. Children with Group A Strep pharyngitis classically have exudative pharyngitis, bilateral tender anterior cervical nodes, and a lack of nasal congestion or cough. Food poisoning (A) may present with vomiting and sometimes fever, but exudative pharyngitis and lymphadenopathy are not typical features. Viral gastritis (C) may also present with vomiting and fever but does not cause exudative pharyngitis or anterior cervical lymphadenopathy. Viral upper respiratory infections (D) are a common cause of pharyngitis and anterior cervical lymphadenopathy. However, viral upper respiratory infections typically present with nasal congestion and cough. Moreover, the typical viruses that cause upper respiratory infection (ex. adenovirus) are more likely to cause diarrhea than vomiting.

A 53-year old previously healthy woman visits her primary care physician to review the results from the biopsy obtained from the solitary 8 mm polyp discovered in her colon by a routine colonoscopy. The report confirms that this is a hyperplastic polyp. Her family history is negative for colon cancer. Which one of the following is the most appropriate interval for follow-up colonoscopy in this patient? 1 year 10 years 3 years 5 years

Correct Answer ( B ) Explanation: Small hyperplastic polyps, which are defined as less than 10 mm in size, are not neoplastic and are considered low risk. For patients at low risk the recommended interval between screening colonoscopies in 10 years. Adenoma polyps are the most common sub-classification of neoplastic polyps and always need more frequent monitoring. Patients with only 1 to 2 small (<1cm) tubular adenomas should undergo repeat colonoscopy in 5-10 years (B). Patients with 3-10 adenomas or any adenoma greater than 1 cm, displaying high-grade dysplasia or with villous features, should undergo a repeat colonoscopy at 3 years (C). The interval for more extensive disease is best individualized but can be as often as annually (A) in unusual cases such as sessile adenomas or a familial syndrome.

A 60-year-old man presents to the office with a history of osteoporosis. Which of the following statements is correct as it relates to men with osteoporosis? Anorexia is a common finding Back pain with vertebral compression is the most common presenting complaint Gonadotropin releasing hormone analogues for prostate cancer lessens the chances of developing osteoporosis Secondary causes includes hyperthyroidism and coronary artery disease

Correct Answer ( B ) Explanation: The clinical presentation of osteoporosis in men is often different from that in women, especially in relation to the time to diagnosis after symptoms begin. Back pain with vertebral compression is the most common presenting complaint. Bone density measurements are less frequently obtained in men, but they are performed after symptoms occur, a far different situation than in postmenopausal women. Lack of androgens results in a skeletal deficit. Peak bone mass is clearly reduced in androgen-insufficient young male patients whether the condition results from idiopathic hypogonadotropic hypogonadism, Klinefelter's syndrome, or constitutional delayed puberty. Anorexia (A) in male patients is extremely rare, and the athlete's triad so characteristic of women runners (i.e., exercise, hypogonadism, and low bone mass) is infrequent in men. The use of long-acting gonadotropin-releasing hormone analogues (C) that block androgen production and are administered in the treatment of prostate cancer is also associated with significant bone loss and fractures in later life. Secondary causes (D) of osteoporosis dominate this disease in men. Hypogonadism and hypercortisolemia are the major etiologic factors in male osteoporosis and must be considered regardless of the phenotypic presentation. However, certain other conditions, including gluten enteropathy, gastric resection or bypass, and ethanol abuse, are more common in men than women with osteoporosis

In normal hearts, which of the following heart valves is composed of two cusps? Aortic Mitral Pulmonic Tricuspid

Correct Answer ( B ) Explanation: The mitral valve is the only heart valve with two cusps. Each cusp is a double layer of endocardium attached at its base to the fibrous skeleton of the heart. The margins of the cusps are attached to muscular projections from the ventricles (papillary muscles) via tendinous cords (chordae tendineae). The aortic (A), pulmonic (C), and tricuspid (D) valves all have three cusps. There are individuals who are born with congenital bicuspid aortic valve which has only two cusps.

A 47-year-old woman with a history of hypertension presents to the ED with a severe headache that began suddenly 3 hours prior to arrival. She has a history of recurrent frontal headaches for the last month and is currently being treated for migraine. Today, she also complains of blurry vision, diplopia, nausea, vomiting, and confusion. Her vital signs are BP 95/45 mm Hg, HR 118, RR 23, T 36.7°C, and pulse oximetry of 98% on room air. Neurologic exam reveals a dilated, minimally reactive left pupil; a globe deviated inferiorly; and bitemporal hemianopsia. Lab results reveal only hyponatremia of 129. Normal saline is administered. A CT scan reveals an intrasellar mass. Which of the following is an appropriate step in the management of this patient? Administer intravenous 3% saline Administer intravenous hydrocortisone Administer intravenous mannitol Arrange for rapid radiation therapy

Correct Answer ( B ) Explanation: The patient has pituitary apoplexy from a pituitary tumor. The initial symptoms of pituitary apoplexy are related to the increased pressure in and around the pituitary gland. The most common symptom, in more than 95% of cases, is a sudden-onset headache located behind the eyes or around the temples. It is often associated with nausea and vomiting. The patient's clinical presentation is also consistent with acute (secondary) adrenal insufficiency due to inadequate ACTH production from the pituitary gland. Adrenal insufficiency manifests in this patient as hypotension, fatigue, abdominal pain, and hyponatremia. It is also associated with hyperkalemia and hypoglycemia. Hydrocortisone is the preferred steroid to administer because it provides both glucocorticoid and mineralocorticoid effects. Clinical improvement is usually seen within a few hours of steroid administration. Intravenous 3% saline (hypertonic saline) (A) is most commonly reserved for cases of acute hyponatremia associated with neurologic abnormalities. Intravenous mannitol is an osmotic agent used as a therapy to temporarily reduce intracranial pressure. The patient's symptoms are due to compression of a pituitary tumor on the optic nerve. Mannitol (C) is not going to relieve this process. Radiation therapy (D) has no role in the acute setting. Treatment is surgical and generally requires transsphenoidal surgery.

An 18-year-old marathon runner presents with diffuse muscle aches and weakness. The symptoms began after he ran his first marathon 3 days ago. His urine is dark and has gross blood on dipstick with 5 RBCs/hpf on microscopy. Which of the following tests is most likely diagnostic for this disorder? Non-contrast CT of the abdomen and pelvis Serum creatine kinase Serum myoglobin Serum potassium

Correct Answer ( B ) Explanation: This patient presents with symptoms consistent with rhabdomyolysis, which can be diagnosed by an elevated creatine kinase (CK). Rhabdomyolysis describes the breakdown of skeletal muscle with release of intracellular contents into the blood stream. These contents include creatine kinase, aspartate transaminase, lactate dehydrogenase, myoglobin, aldolase and electrolytes. Muscle breakdown leads to swelling, edema, increased compartment pressure and ischemia. The entry of intracellular components into the bloodstream can lead to renal impairment, insufficiency and irreversible damage. There are four major etiologies of rhabdomyolysis: 1) impaired muscle ATP production, 2) disruption of nutrient delivery to skeletal muscle, 3) increased metabolic demands of skeletal muscle and 4) direct muscle damage. Patients typically complain of swelling and tenderness of muscle groups as well as cramping and pain. Tea-colored urine often prompts physician evaluation. The disease is commonly suspected by the presence of blood on dipstick with the absence or only minimal amount of RBCs on microscopy. The positive dipstick is the result of myoglobinuria. Elevated CK levels are diagnostic of rhabdomyolysis. The enzyme rises over 12 hours and stays elevated for 5 days. Treatment of rhabdomyolysis focuses on identifying compartment syndrome, correcting electrolyte disorders and fluid resuscitation to correct renal impairment if present. A non-contrast CT of the abdomen and pelvis (A) is indicated for the diagnosis of urolithiasis, which presents with hematuria but this patient does not have a clinical scenario consistent with this disease. Serum myoglobin (C) is useful in the early presentation of rhabdomyolysis but it returns to normal within 24 hours. Serum potassium (D) may be elevated in rhabdomyolysis as the result of renal impairment and mycocyte breakdown but is not diagnostic.

An unimmunized 7-year-old boy without any previous medical history presents with fever and cough. The parents report that he has been coughing for 2 weeks. The patient has sudden fits of coughing followed sometimes by post-tussive emesis. Which of the following is an appropriate plan? Albuterol nebulizer Corticosteroid therapy Macrolide antibiotic Respiratory viral panel

Correct Answer ( C ) Explanation: Although a vaccine for pertussis was developed in the 1940s, it remains a cause of significant respiratory illness in the US each year. Decreasing immunity in adults has contributed to persistent annual cases of this disease. The disease occurs most commonly in children under the age of one who have not received the entire vaccine series and nonimmunized children. Pertussis is caused by Bordatella spp. which specifically adheres to the ciliated epithelial cells of the respiratory tract. There are three phases to the illness. The catarrhal phase lasts 1-2 weeks after an incubation period of 7-10 weeks. Patients are most infectious during this phase, the symptoms of which are indistinguishable from an average upper respiratory infection. Phase two is the paroxysmal phase characterized by the resolution of fever and increased cough. For 2-4 weeks, patients have paroxysms of the classic "staccato" cough. Patients cough in fits followed sometimes by the "whoop" when they forcefully inhale. Post-tussive emesis, syncope and brief apneic episodes may occur. The convalescent third phase lasts for weeks to months and is characterized by a residual cough. Antibiotic therapy with a macrolide is indicated to decrease the infectivity of patients. The use of antibiotics does not decrease the duration of illness or severity of illness. The primary treatment is supportive care. Erythromycin is contraindicated for infants < 1 month due to increased risk for pyloric stenosis. Infants < 1 month old should receive azithromycin. Albuterol therapy (A) does not have any effect on the frequency or severity of coughing episodes. In patients with underlying reactive airway disease having an exacerbation because of infection with pertussis, beta-agonist therapy may have benefit. Corticosteroid therapy (B) has not shown widespread benefit in the treatment of pertussis. In a subset of critically ill younger patients, there may be some benefit from their use. A respiratory viral panel (D) is sometimes used to help cohort patients requiring admission to the hospital. Pertussis is a bacterial infection and will not show up on most respiratory panels.

A 14-day-old neonate presents to clinic with bilateral eye discharge and conjunctival injection. What treatment is indicated? Ceftriaxone intramuscular Erythromycin ointment Oral erythromycin Warm compresses and lacrimal duct massage

Correct Answer ( C ) Explanation: Eye discharge is extremely common amongst neonates. The most common cause of neonatal eye discharge is lacrimal duct obstruction. However, accompanying conjunctival involvement indicates a probable infectious etiology. Chlamydia trachomatis is the most common cause of neonatal conjunctivitis. The organism is acquired during delivery and has an incubation period of five to fourteen days. Thus, any conjunctivitis occurring less than five days from the time of delivery is unlikely to be caused by C.trachomatis, and evaluation for alternative etiologies should be pursued. Often the first manifestation of the Chlamydial conjunctivitis is watery discharge, which later becomes mucopurulent. Conjunctival chemosis then develops, and the conjunctiva may become so friable that they may bleed. Eyelid swelling may also develop. The treatment of choice of Chlamydial conjunctivitis is oral erythromycin. After completion of treatment, most neonates have full resolution of symptoms. Untreated neonates may develop permanent corneal and conjunctival scarring. Ceftriaxone (A) is the treatment of choice for neonatal gonococcal conjunctivitis. In contrast to the conjunctivitis caused by C.trachomatis, gonorrheal conjunctivitis occurs almost exclusively in the first few days of life. The discharge of gonococcal conjunctivitis is copious, and the eyelids may be so swollen that the conjunctiva cannot be visualized. Erythromycin ointment (B) is not as effective as oral erythromycin in the treatment of chlamydial conjunctivitis. Moreover, erythromycin ointment does not eradicate nasopharyngeal colonization with C.trachomatis. Warm compresses and lacrimal duct massage (D) are the treatments of choice for lacrimal duct obstruction. However, the patients conjunctival involvement warrants evaluation and treatment for infectious conjunctivitis.

Which of the following ophthalmologic devices aids in the diagnosis of glaucoma? Phoropter Slit lamp Tonometer Wood's lamp

Correct Answer ( C ) Explanation: Glaucoma, the leading cause of blindness in the world and the second most common cause of blindness in the US, refers to a secondary optic neuropathy. It may or may not be due to increased intraocular pressure. Increased intraocular pressure is largely due to poor aqueous humour flow between the anterior and posterior eye chambers through the trabecular meshwork. There are four main types. Congenital glaucoma is rare. Secondary glaucoma occurs after trauma or injury. Closed-angle glaucoma, an ophthalmologic emergency, usually presents as an acute loss of vision, feeling of eye swelling, cloudy or steamy vision with halos around lights and eye pain. Open-angle glaucoma, the most common type, is a chronic, slowly progressive disease of painless loss of vision, in which most patients have no symptoms until late in the disease. Diagnosis requires tonometry testing, in which the intraocular pressure is measured directly. Visual loss can be prevented if glaucoma is diagnosed early, as such, regular ophthalmological exams are important. Treatment options include eye drops, oral medications, laser procedures or surgery. A phoropter (A) is used during a visual examination to determine refractive error, which is then used to determine an individual's eyeglass prescription ("which is better, 1 or 2, 3 or 4..."). A slit lamp (B) is used with fluorescein stain to detect corneal abrasions, not elevated intraocular pressure. A Wood's lamp (D) uses ultraviolet light to detect corneal abrasions, epithelial bacterial or fungal infections and vitiligo.

A 17-year-old man presents after being thrown a far distance off of a horse. Which of the following is consistent with an anterior cord syndrome? Isolated motor function loss Loss of all motor and sensory function Loss of pain and temperature, loss of motor function Upper greater than lower motor weakness

Correct Answer ( C ) Explanation: In order to fully understand the different syndromes of injuries to the spinal cord, it is imperative to understand the location of the tracts of the cord. The posterior columns carry tracts responsible for ipsilateral position and vibratory sensation. The lateral spinothalamic tract carries fibers for contralateral pain and temperature. The lateral corticospinal tract is responsible for ipsilateral motor function. Syndromes may be incomplete depending on how much of the cord is affected by the injury. In the anterior spinal cord syndrome, just the posterior columns are preserved and so patients lose all pain and temperature sensation as well as motor function. Most cases of anterior cord syndrome follow aortic surgery, but it has also been reported in the setting of hypotension, infection, vasospasm, or anterior spinal artery ischemia or infarct. In trauma, typically hyperflexion of the cervical spine causes the injury to the spinal cord. Loss of all motor and sensory function (B) occurs with a complete transection of the spinal cord. Most commonly this occurs after a significant trauma. Isolated motor function loss (A) is not a classic syndrome and would result from a small area of injury on the cord just involving the corticospinal tract. Upper greater than lower motor weakness occurs (D) with a central cord syndrome. Sensory involvement is variable although burning dysesthesias in the upper extremities may occur. Most commonly the syndrome occurs after a fall or motor vehicle accident. Anterior cord may also be known as ventral cord syndrome.

A 68-year-old man was admitted to the hospital for an acute exacerbation of his chronic systolic congestive heart failure. What education should be given to this patient upon discharge to help prevent readmission? Avoid physical activity Elevate lower extremities Monitor daily weights Restrict fluid intake

Correct Answer ( C ) Explanation: Instructing patients to monitor daily weights can help prevent heart failure readmission. Heart failure is one of the most common causes of hospitalization, hospital readmission and death. Due to the complexity and long-term nature of heart failure regimens, the need for careful diet and weight management, and the importance of intervention in the early phases of decompensation, patient self-management is crucial in avoiding hospitalizations. Patients should be specifically instructed to take all medications as directed, monitor daily weights, monitor for signs and symptoms, adhere to a low-sodium diet, limit alcohol consumption and stop smoking. Elevation of the lower extremities (B) can help reduce edema but it will not help prevent readmission to the hospital with a congestive heart failure exacerbation. Heart failure patients should not be advised to avoid physical activity (A). In stable patients, increasing physical activity or regular exercise can help diminish symptoms. Lastly, restricting fluid intake (D) has not been shown to prevent heart failure readmissions.

A previously healthy 18-year-old woman presents to the emergency department with complaints of fever, headache, and neck stiffness. She is accompanied by her sister, who expresses concern because the patient seems suddenly confused and cannot remember what she did yesterday. After you administer empiric intravenous antibiotics, which of the following is the next best step? Chest X-ray Complete blood count with differential Lumbar puncture Urinalysis

Correct Answer ( C ) Explanation: Meningitis is an inflammation of the tissues surrounding the brain and spinal cord (meninges) and may be of infectious (bacterial, viral, or fungal) and various other etiologies. The classic clinical manifestations include nuchal rigidity, fever, and altered mental status. Patients often present with headache as well. All patients with suspected meningitis should have lumbar puncture (LP) to evaluate the cerebrospinal fluid (CSF) unless this procedure is contraindicated. There are no absolute contraindications to LP. Relative contraindications include patients with evidence of increased intracranial pressure, thrombocytopenia, bleeding diathesis, or spinal epidural abscess. Acute bacterial meningitis is a medical emergency and left untreated or treated late is almost universally fatal. Treatment involves addressing systemic complications and initiating empiric antibiotic therapy as soon as possible. Up to half of patients with pneumococcal meningitis may have evidence of pneumonia on chest X-ray (A), but this is not part of the initial workup of patients with suspected meningitis. Complete blood count with differential (B) is often ordered in the workup for bacterial meningitis and generally shows increased white blood cell count but is not as important to order initially as an LP. Urinalysis (D) is generally not a helpful test in the diagnosis of meningitis and therefore not recommended in the workup for patients with suspected bacterial meningitis.

What is the most common sexually transmitted infection in the United States? Chlamydia Gonorrhea Human papillomavirus Syphilis

Correct Answer ( C ) Explanation: More than 50% of sexually active individuals will contract human papillomavirus (HPV) at some point in their lives. There are over 100 types of HPV, many of which are asymptomatic and unrecognized. Two high-risk types of the virus, HPV 16 and 18, are known to cause cervical and anogenital cancers in men and women. HPV 6 and 11 are low-risk types that cause genital warts. Two HPV vaccines are available and are recommended for both men and women aged 11-12 years old. HPV2, Cervarix® for women only and HPV4, Gardasil® for both men and women. The vaccinations can also be given to females ages 13-26 years and males ages 13-21 years who were not vaccinated earlier. Of all diseases that are reportable to the Centers for Disease Control, chlamydia (A) is the most common and gonorrhea (B) is the second most common. However, HPV is the overall most common sexually transmitted disease. Both chlamydia and gonorrhea are major causes of pelvic inflammatory disease in women. Syphilis (D) rates are lower than those of chlamydia, gonorrhea and HPV, but have been increasing since 2000. Syphilis infection is considered to be a major health problem among men who have sex with men and there is a high rate of co-infection with HIV in this population.

A full-term neonate is brought to the ED for constant crying for the last three hours. In the ED, he sleeps quietly in his mother's arms. He cries when you examine him but is immediately consoled when he is swaddled and held. His exam is normal. What is the next most appropriate step in his management? CT scan of the head to rule out intracranial hemorrhage Lumbar puncture to rule out serious bacterial infection Reassurance Skeletal survey for abuse

Correct Answer ( C ) Explanation: Neonates cry in varying patterns throughout the day. Although crying is typically a sign of hunger or normal uncomfortable states (wet diaper, gas), it may also signal pain or underlying disease. An easily consoled infant without a source of crying after a thorough history and physical exam can be discharged with parental reassurance. A thorough exam for crying includes completely exposing the infant; palpating the fontanelles; fluorescein staining of the eyes for corneal abrasions; examining the fingers, toes, and penis for hair tourniquets; and checking for hernias. It is also important to look for retinal hemorrhages and palpate the extremities for accidental or deliberate trauma. Intracranial hemorrhage (A) may be caused by neonatal alloimmune thrombocytopenia, birth trauma, vitamin K deficiency, or acute injury. Symptoms of intracranial hemorrhage are irritability, inconsolable crying, poor feeding, and lethargy. Signs include a bulging fontanelle, retinal hemorrhages, and bruising. Serious bacterial infection (SBI) (B) may be present in any neonate but will usually present with more than just crying. The patient may have feeding difficulties, jaundice, hypo- or hyperthermia, tachypnea, or respiratory distress. Neonates with suspected sepsis must undergo a full septic workup with blood cultures, urine cultures, and CSF evaluation. They should be started on empiric antibiotics until cultures are negative. Neonatal child abuse (D) may be difficult to diagnose. Signs to look for include those correlating with underlying intracranial hemorrhage (see explanation for choice A), bruising, or deformity. Appropriate authorities must be contacted if child abuse is suspected, even if it is not confirmed.

What class of medication should be used for primary prophylaxis to prevent variceal hemorrhage? ACE inhibitor Antihistamine Beta blocker Proton pump inhibitor

Correct Answer ( C ) Explanation: Nonselective beta blockers are used for the primary prophylaxis of variceal bleeding and have demonstrated a decrease in the risk of variceal bleeding. All patients with large varices (diameter greater than 5 mm) should be considered for prophylactic therapy ("primary prophylaxis") to prevent variceal bleeding. The presence of additional endoscopic signs such as red wales does not influence the decision regarding prophylactic therapy. The mortality rate is reduced from 28.4% in control patients to 23.9% in patients taking a beta blocker; the absolute risk reduction is 4.5%. The number of patients needed to be treated to prevent one death is approximately 22. In patients who do not bleed during therapy and who do not experience side effects, treatment should be continued indefinitely because withdrawal of a beta blocker can result in an increased risk of bleeding. A long-acting preparation of propranolol or nadolol may be started; the usual starting dose of long-acting propranolol is 60 mg once daily and that of nadolol is 20 mg once daily. Because the risk of bleeding is greatest at night, the beta blocker should probably be administered in the evening. ACE inhibitors (A) and antihistiamines (B) play no role in the pre-prophylaxis or primary prophylaxis of esophageal varices. Although proton pump inhibitors (D) may ease symptoms of gastritis they will not alter the risk of esophageal variceal bleeding.

An adult patient presents with itching and a feeling fullness in the left ear for the past month. The patient states that this all began a few weeks after he was successfully treated for a sinus infection with amoxicillin. Exam shows a mildly erythematous ear canal with thickened, white, creamy exudate. There is slight discomfort with pulling on the pinna. After performing simple debridement, which of the following is the most appropriate treatment to start this patient on? Antipyrine/benzocaine otic Cephalexin oral Clotrimazole topical Neomycin/polymyxin otic

Correct Answer ( C ) Explanation: Otomycosis is a fungal infection of the external auditory canal. Otomycosis can occur as the primary infection or as a result of antibiotic therapy for a bacterial infection. Swimming, ear canal scratching and cleaning also predispose patients for otomycosis. Patients with otomycosis most commonly report ear pruritus, discomfort, discharge, and a feeling that something is in the ear canal. Pain and edema are less severe than with bacterial external otitis. Fine fungal filaments and spores may be seen on exam. Treatment typically includes antifungals, such as clotrimazole, or combination mixtures, such as 1:1 mixture of vinegar & water. Antipyrine/benzocaine otic (A) drops are primarily used as an analgesic and sometimes used to facilitate cerumen removal. These would treat the patient's pain but would not address the infection appropriately. Oral antibiotics, such as cephalexin (B) would only be necessary for severe external otitis or the presence of cellulitis extending beyond the ear canal, and therefore has no benefit for this presentation. Because this presentation appears fungal, antibiotic drops such as neomycin/polymyxin (D) would be ineffective.

A 16-year-old girl returns from a summer abroad volunteering at an HIV clinic in Zimbabwe. She is going to volunteer at her local hospital, which requires tuberculosis skin testing. Her skin test is read as 14 mm in diameter. Chest X-ray is negative and she is asymptomatic so she is started on a 9-month course of isoniazid. What vitamin should she concurrently be started on? Cobalamin Niacin Pyridoxine Thiamine

Correct Answer ( C ) Explanation: Pyridoxine (vitamin B6) supplementation is recommended when taking isoniazid because isoniazid alters the activity of vitamin B6. The tuberculosis skin test is measured 48 to 72 hours after placement. A positive test is induration of 5 mm or more in patients with HIV or other immunocompromising conditions, 10 mm or more in patients younger than age 4, with certain medical conditions (e.g. diabetes, renal failure), in health care workers or in others with high-risk exposure, and 15 mm or more in patients with no risk factors. Patients with positive skin tests with a negative chest X-ray and no symptoms should be started on treatment with isoniazid for latent tuberculosis. Cobalamin (vitamin B12) (A) is important for DNA synthesis and deficiency can lead to pernicious anemia and abnormal neurological symptoms. Niacin (vitamin B3) (B) is used to treat hypercholesterolemia and pellagra. Thiamine (vitamin B1) (D) deficiency can cause neurological symptoms as well as seen in Wernicke's encephalopathy and Korsakoff syndrome.

Which of the following is a medical treatment for congenital long QT syndrome? Flecainide Propafenone Propranolol Quinidine

Correct Answer ( C ) Explanation: Propranolol is the appropriate treatment for congenital long QT syndrome as beta blockers are the mainstay of treatment for this condition. Long QT syndrome is the most common channelopathy and is thought to affect 1 in 5000 persons. It is a disorder of myocardial repolarization characterized by a prolonged QT interval on ECG and an increased risk of sudden cardiac death: QTc is usually > 460 ms. This syndrome is associated with an increased risk of a characteristic life threatening polymorphic ventricular tachycardia known as torsades de pointes or "twisting of the points." The primary symptoms in patients with long QT syndrome include palpitations, syncope, seizures, and cardiac arrest, and patients usually have syncope related to polymorphic ventricular tachycardia. Factors conferring the highest risk for sudden cardiac death include a history of sudden cardiac arrest, recent syncope, and QTc interval greater than 500ms. Beta blockers, such as propranolol, have shown to reduce both syncope and sudden cardiac death. The goal of beta blocker therapy is to blunt the maximal heart rate achieved during exertion and are thought to interrupt the "trigger" for torsades de pointe and shorten the QT interval. Flecainide (A) and propafenone (B) are both class IC III antiarrhythmics which are commonly used for atrial and ventricular dysrhythmias; however, they can lengthen the QT interval. Quinidine (D) is a class IA antiarrhythmic that can also lengthen the QT interval.

A 23-year-old man presents with fever, headache, and myalgias ten days after hiking in the woods in South Carolina. Vital signs are BP 110/70 mm Hg, HR 105 beats/minute, RR 18 breaths/minute, and temperature 102 °F. On physical examination, he has right upper quadrant abdominal tenderness and scleral icterus. What is the most appropriate therapy? Amoxicillin 500 milligrams orally three times daily Ceftriaxone 1 grams intravenously every 12 hours Doxycycline 100 mg orally two times daily Rifampin 600 mg orally once daily

Correct Answer ( C ) Explanation: This patient presents with signs and symptoms consistent with human ehrlichiosis, a tick-borne illness seen primarily in the South Central and South Atlantic United States. The peak incidence of the disease is typically the summer months (June-August). There are multiple members of the Ehrlichia family; however, they are all gram-negative, obligate intracellular coccobacilli that reside within circulating leukocytes. Signs and symptoms begin approximately nine days after discovery of the tick and 90% of patients recall the bite. Signs and symptoms include abrupt onset of fever, headache, myalgias, and rigors with less frequently associated GI symptoms. Complications, although uncommon, include optic neuritis, ARDS, meningitis, pericarditis, renal failure, and DIC. Laboratory studies characteristically reveal leukopenia, thrombocytopenia, and elevated liver function tests. Management of ehrlichiosis is doxycycline 100 mg two times daily for 7-14 days, regardless of age. The vast majority fully recover without any residual morbidity. Amoxicillin 500 mg PO three times daily (A) is the appropriate treatment for Lyme disease in pregnant patients or children younger than 8 years of age. Ceftriaxone 1 g IV every 12 hours (B) is the appropriate treatment for a patient with neurologic or cardiac manifestations of Lyme disease. Rifampin 600 mg PO once daily (D) is the appropriate treatment for patients with ehrlichiosis who have a life-threatening allergic reaction to doxycycline or other tetracyclines.

A 55-year-old man presents to your office with gradual onset atraumatic low back pain, leg pain, and numbness. The pain is most often bilateral, involving the buttocks and thighs and spreads distally towards the feet. The leg pain begins and worsens with walking and standing and is relieved by sitting or lying down with hips and knees drawn up in a sitting posture. Which of the following is the most likely diagnosis? Ankylosing spondylitis Scoliosis Spinal stenosis Spondylolisthesis

Correct Answer ( C ) Explanation: Spinal stenosis is the narrowing of the spinal canal and neural foramina. The more common acquired spinal stenosis is caused by degenerative changes in the intervertebral disks, ligaments, and facet joints surrounding the lumbar canal. These degenerative changes can be caused by disk or joint disease, back surgery, and repetitive trauma. Stenosis initially becomes symptomatic at 40 to 50 years and older. Congenital narrowing of the spinal canal causes symptoms earlier in life and is uncommon. Spinal stenosis usually occurs at cervical and lumbar segments. Patients with cervical stenosis present with radiating arm pain, numbness, paresthesia, and motor weakness. The common symptoms with lumbar stenosis are gradual onset low back pain, leg pain, and numbness. The pain is most often bilateral, involving the buttocks and thighs and spreading distally toward the feet. The classic presentation is radiating leg pain (burning or cramping) that begins or worsens with walking and standing and is relieved by sitting or lying down with hips and knees drawn up in a sitting posture (neurogenic claudication). Bending forward diminishes pain. Physical examination is frequently normal but may include loss of lumbar lordosis, impairment of spinal mobility, asymmetric knee or ankle reflexes, and muscle weakness. Results of straight-leg raising are characteristically negative. Advanced imaging studies are obtained to establish and confirm the diagnosis of spinal stenosis when surgery is considered. Plain spine radiographs often reveal degenerative changes. MRI is currently the preferred modality, followed by CT scan. Ankylosing spondylitis (A) usually affects men (male/female ratio 5:1) in their 20s and 30s. It often manifests as vague, somewhat diffuse low back pain, felt generally in the buttocks or sacroiliac area but often in the lumbar area. Pain becomes more persistent and bilateral. Back stiffness after inactivity, such as on awakening, becomes more predominant and is relieved by activity or a hot shower. Idiopathic scoliosis (B) affects women more frequently then men, and is often asymptomatic in young people. The patient with scoliosis may present with apparent deformity, pain, symptoms of difficult movement, or neurologic deficits. Most people with spondylolysis or spondylolisthesis (D) are asymptomatic. For the symptomatic minority, the most common complaint is low back pain, which can range from mild to severe. Back pain is frequently described as a dull, aching pain in the back, buttocks, and posterior thigh, the characteristics of which are not distinct compared with other degenerative back disorders.

A 63-year-old man with a history of hepatitis C presents to the ED with complaints of generalized abdominal pain and distension. Vital signs are HR 110 beats per minute, RR 22 per minute, BP 130/67 mm Hg, T 39.8°C, and oxygen saturation 97% on room air. On exam, his abdomen is tender and moderately distended. Diagnostic paracentesis is performed. Which of the following results should prompt treatment with antibiotics? Ascitic fluid pH of 7.35 WBC of 275 cells/mm3 with 20% lymphocytes WBC of 370 cells/mm3 with 90% neutrophils WBC of 500 cells/mm3 with 40% neutrophils

Correct Answer ( C ) Explanation: Spontaneous bacterial peritonitis (SBP) is an acute bacterial infection of ascitic fluid in patients with liver disease. Patients with SBP lack an apparent external or intra-abdominal focus of infection. Although, by definition, ascites must be present for SBP to develop, free peritoneal fluid may not always be clinically apparent. Definitive diagnosis is made by culture of the ascitic fluid, but treatment decisions should be made prior to culture results. An ascitic fluid granulocyte count >500 correlates with positive cultures in more than 90% of cases; however, ED treatment for SBP should be initiated if the neutrophil count is greater than 250. A pH of <7.34 (A) or a pH gradient between arterial blood and ascitic fluid >0.10 is another reliable early indicator of SBP. The presence of 20% lymphocytes with 275 WBCs (B) correlates with 220 neutrophils, and 40% neutrophils with 500 WBCs (D) correlates with 200 neutrophils, both of which fall below the treatment threshold for SBP.

A 63-year-old man is brought to the ED by paramedics secondary to aphasia that began 30 minutes prior to arrival. The man is unable to give any history and has never been to your hospital. His vital signs are T 37°C, BP 180/98, HR 90, and RR 20. His exam is unremarkable except for profound expressive aphasia. Capillary blood glucose is 124 mg/dL. His ECG shows normal sinus rhythm with normal ST segments and T waves. CT scan of the head is normal. Sixty minutes after arriving to the ED, the patient's aphasia resolves. Which of the following are the most appropriate diagnosis, treatment, and disposition for this patient? Hypertensive emergency; start IV antihypertensive therapy and admit to the medical intensive care unit Stroke; administer tissue plasminogen activator; admit to stroke unit Transient ischemic attack; administer aspirin and dipyridamole; admit to stroke unit Transient ischemic attack; administer low molecular weight heparin and warfarin; discharge home with close neurology follow up Transient ischemic attack; start heparin infusion; admit to stroke unit

Correct Answer ( C ) Explanation: The 2009 AHA/ASA guidelines define transient ischemic attack (TIA) as "a transient episode of neurological dysfunction caused by focal brain, spinal cord, or retinal ischemia without acute infarction." Previously, TIAs were differentiated from strokes by the duration of symptoms—with TIAs lasting less than 24 hours. However, it has since become evident from imaging studies that many patients with symptoms lasting less than 24 hours have had a stroke. The latest class IA treatment recommendation is for aspirin alone or aspirin plus dipyridamole. Patients diagnosed with a TIA should be admitted for assessment of stroke risk factors because the 90-day risk of subsequent stroke is ~ 9.5%. The risk of stroke increases in patients older than 60 years, with a history of hypertension or diabetes mellitus, and whose symptoms last > 10 minutes or are associated with weakness or speech impairments. Even though hypertension is a risk factor for TIA (A) or ischemic stroke, and can cause cerebral impairment, the AHA/ASA recommends permissive hypertension if the patient is not receiving thrombolytics. In such cases, blood pressure should be acutely lowered only if it remains persistently > 220/120 mm Hg with a goal of reducing by 15% within 24 hours. This patient's symptoms have resolved (B), so he is not a candidate for thrombolysis. Unfractionated and low molecular weight heparin (E) have not been shown to improve outcomes in stroke patients and increase the risk for intracerebral hemorrhage. Warfarin (D) is typically prescribed to patients with atrial fibrillation to reduce the risk of stroke from a thrombotic embolism. It should not be started until a full stroke workup has been performed.

What is the most appropriate first line treatment for plantar fasciitis? Corticosteroid injection Extracorpeal shock wave ultrasound NSAIDs Surgical therapy

Correct Answer ( C ) Explanation: The plantar fascia is a tough layer of the sole that is functionally significant during foot strike and the early stance phase of walking. Plantar fasciitis is an overuse injury of insidious onset that usually begins with pain on first weight bearing in the morning or after prolonged sitting. This progresses to persistent pain during gait. Pain and tenderness are localized to the medial aspect of the heel. Plain radiography is not diagnostic but shows a calcaneal spur in 50% of patients with plantar fasciitis. NSAIDs are considered first line treatment, along with rest, ice, and shoe inserts. Corticosteroid injection (A) is considered if conservative therapy fails. Pain relief has only been demonstrated at 1 month with no significant difference at 6 months. Extracorpeal shock wave ultrasound (B) is a second-line therapy with mixed results for any positive benefit. Surgical therapy (D) is considered if all other measures fail. There is no proven benefit that surgical therapy is any more beneficial than conservative therapy.

What is a principal risk factor for peptic ulcer disease? Alcohol consumption Emotional stress Helicobacter pylori infection High fat diet

Correct Answer ( C ) Explanation: The principal risk factors for peptic ulcer disease are Helicobacter pylori infection and NSAID use. H. pylori is a gram-negative bacillus that is uniquely adapted to life in the stomach. It is a major cause of peptic ulcer disease and accounts for a large proportion of peptic ulcers in countries where H. Pylori infection is highly prevalent, such as in Asia. In the United States and Western Europe, the original estimates that H. pylori infection was the cause of 90% or more of duodenal ulcers, and 60% or more of all gastric ulcers have been lowered by the declining prevalence of H. pylori infection. It is estimated that close to 70% of duodenal ulcers are related to H. pylori in Western populations. For example, in an analysis of patients who participated in H. pylori eradication trials in the United States, the proportion of patients with peptic ulcer disease and H. pylori infection was 73%. The role of alcohol (A) remains uncertain. Alcoholic beverages stimulate gastric acid production. Moreover, direct application of high concentrations of alcohol to the gastric mucosa causes demonstrable mucosal injury. With regard to diet (D), for example, ulcer prevalence rates differ considerably in the north and south of India, where the principal cereal is wheat versus rice, respectively. However, many other potential confounders were not accounted for in these populations. An association between the ingestion of spicy foods and peptic ulcer disease is weak, at best. Emotional stress (B) was proposed as a major cause of ulcer disease or as a precipitant of ulcer complications, and much was written about its relationship to peptic ulcer disease prior to the description of H. pylori.

34-year-old woman presents to the Emergency Department five days after total thyroidectomy with tingling in her hands. On physical examination, she has twitching at the corner of her mouth upon tapping the side of her face. Which of the following laboratory abnormalities is most likely? Hyperkalemia Hypervitaminosis D Hypocalcemia Hypophosphatemia

Correct Answer ( C ) Explanation: This patient is exhibiting signs and symptoms of hypocalcemia, most commonly caused by hypoparathyroidism. Signs and symptoms are exclusively related to hypocalcemia, and include reduced myocardial contractility, perioral and distal extremity paresthesias, and tetany. There are two hallmark physical exam findings with significant hypocalcemia: Chvostek's sign and Trousseau's sign. Chvostek's sign consists of facial twitching caused by tapping the facial nerve anterior to the ear. Trousseau's sign consists of carpal spasm when a blood pressure cuff is inflated on the upper arm above the systolic pressure for greater than three minutes. Partial or total thyroidectomy is the most common etiology of hypoparathyroidism as the parathyroid is located immediately behind and is often partially or totally attached to the thyroid gland. Other causes of hypoparathyroidism include a congenital deficiency or infiltrate of the parathyroid glands from metastatic carcinoma, hemochromatosis, or Wilson disease. Diagnosis of hypocalcemia is often made on physical exam; however, laboratory testing should be obtained including both a total and ionized calcium level, serum 25-(OH) and 1,25-(OH) vitamin D, and parathyroid hormone levels. An ECG should also be performed to evaluate for prolongation of the QT interval. Management includes calcium and vitamin D supplementation. In severe, symptomatic hypocalcemia, calcium gluconate or calcium chloride should be administered intravenously, depending on the severity of symptoms. Calcium chloride has three times more calcium per equal volume, but has the risk of tissue ischemia and necrosis if infused peripherally. Thus, calcium gluconate is preferred. Hyperkalemia (A) may cause muscle cramps, but is not typically associated with tetany or paresthesias. Hypoparathyroidism is also not classically associated with hyperkalemia. Hypoparathyroidism is commonly associated with hypovitaminosis D and hyperphosphatemia as opposed to hypervitaminosis D (B) and hypophosphatemia (D).

A 6-year-old boy presents with ankle pain after getting kicked while playing soccer. He has tenderness anteriorly along the ankle joint. An X-ray is shown above. The patient has significant pain when walking. Which of the following is the most appropriate management? Analgesics and follow up in 2 weeks CT scan of the ankle Immobilization of the ankle and non-weight bearing Immobilization of the ankle and weight bearing as tolerated

Correct Answer ( C ) Explanation: This patient presents with signs and symptoms concerning for a Salter-Harris Type I fracture and should be immobilized and made non-weight bearing. Children's bones, unlike those of adults, contain cartilaginous centers near the end of the bone that give rise to new bone growth (epiphysis). Because these areas are radiolucent, they are not visualized on radiographs. Injuries to the epiphysis may result in abnormal bone growth if they do not heal. Therefore, when injury to this area is suspected, conservative management with splinting and non-weight bearing (if lower extremity) status is recommended to promote healing and discourage worsening injury. Injuries to these areas are referred to as Salter-Harris Type I injuries (displacement of the epiphysis). The Salter-Harris classification system was designed to aid in the description and management of pediatric fractures. Analgesics (A) may be required but the patient should be made non-weight bearing and splinted prior to discharge. CT scan of the ankle (B) is not necessary as it would not change treatment at this point. Immobilization is vital but the patient should not be allowed to place weight on the extremity (D).

A three-year-old boy weighing 15 kg requires repair of a laceration to his forearm. What is the maximum amount of 1% lidocaine without epinephrine that can be used for analgesia during the repair? 10.5 ml 3.75 ml 7.5 ml 8 ml

Correct Answer ( C ) Explanation: Wound anesthesia is an important consideration prior to beginning laceration repair on children. Lidocaine, an amide anesthetic, is an excellent agent capable of providing local anesthesia during laceration repair, foreign body removal, abscess management, and line insertion. There are multiple forms and concentrations of lidocaine. The maximum weight based dose of lidocaine without epinephrine is 5 mg/kg. The maximum dose of lidocaine with epinephrine is 7 mg/kg. This higher toxic dose is due to the vasoconstriction caused by epinephrine, which leads to less systemic absorption of lidocaine. Lidocaine toxicity affects the CNS, cardiovascular, and hematologic systems. For a patient that weighs 15 kg, the maximum dose of 1% lidocaine is is 75 mg (5 mg x 15 kg = 75 mg). Because the question asked for the amount, not the dose, it is necessary to know how many mg/mL are present in a given solution. A 1% solution of lidocaine has 10 mg/mL. Therefore, 75 mg (maximum dose) / 10 mg/mL = 7.5 mL. So 7.5 mL of 1% lidocaine is the maximum amount that can be used for anesthesia in this patient. If a 1% lidocaine with epinephrine solution is used, the amount of volume that can safely be administered is 10.5 mL (7 mg x 15 kg / 10 mg/mL = 10.5 mL). 3.75 mL (B) would be the correct maximum dose for a clinician using a 2% lidocaine solution. 10.5 mL (A) would be correct if the question had asked for lidocaine with epinephrine (as this has a higher maximum dose of 7 mg/kg). 8 mL (D) is not the correctly calculated dose for any formulation of injectable lidocaine in a 15 kg child.

A 60-year-old man who has a 20-pack-year smoking history presents to his pulmonologist for an evaluation of his COPD. He recently quit smoking 5 years ago and underwent a colonoscopy 8 years ago. He received his pneumococcal vaccine four years ago and his influenza vaccine this year. Which of the following should be offered to this patient? A second dose of the pneumococcal vaccine Annual screening for lung cancer with low-dose computed tomography scan Tuberculin skin testing Zoster vaccine

Correct Answer ( D ) Explanation: A single dose of the zoster vaccine is recommended for adults aged 60 regardless of whether they report a previous episode of herpes zoster. A person who reports a negative history of varicella can still receive the zoster vaccine. The zoster vaccine reduces the risk of herpes zoster (shingles) by approximately 70%. The vaccine suppresses reactivation of the varicella zoster virus in the dorsal root ganglia. Pneumonia is one of the leading causes of morbidity and mortality in older adults and therefore the pneumococcal vaccine is recommended for healthy adults aged 65 years or older. A one-time revaccination (A) is recommended for adults if they were vaccinated greater than 5 years previously for a high -risk condition and they were younger than 65 years of age at that time. The pneumococcal vaccine is given as a series, beginning at 2 months of age to all infants and also beginning at 2 years of age for high- risk populations such as those with chronic lung disease, asplenia, malignancy, and chronic cardiac disease. The pneumococcal vaccine is given to anyone who has undergone a splenectomy. Annual screening for lung cancer with low-dose CT scan (B) is recommended in adults aged 55-80 years who have a 30 pack-year smoking history and currently smoke or who have quit smoking within the past 15 years. This patient has a 20 pack-year smoking history and therefore does not meet the criteria. However he should be counseled on smoking cessation at this visit. The tuberculin skin test (C) is used to evaluate for delayed hypersensitivity response in order to diagnose prior exposure to tuberculosis. Tuberculin skin testing is recommended for those patients who are at high -risk for tuberculosis. Examples of high- risk populations include those who are immunosuppressed, healthcare workers, immigrants, the homeless, intravenous drug users, prisoners, or those who have been in close contact with others who have active tuberculosis. Different cutoffs in size of induration are used to define a positive test depending on the patient's risk factors.

Which of the following organisms is associated with pneumonia and bullous myringitis? Bordetella pertussis Haemophilus influenzae Streptococcus bovis Streptococcus pneumoniae

Correct Answer ( D ) Explanation: Although bullous myringitis is described as a classic finding in M. pneumoniae infections, it is not specific for mycoplasmal infection and is present in only a few cases. S. pneumoniae infection is most likely the cause of bullous myringitis in the setting of pneumonia. S. pneumoniae is a common cause of community acquired pneumonia. Bullous myringitis is an inflammation of the eardrum in which painful, fluid filled vesicles form. Patients with S. pneumoniae will present with typical signs of pneumonia. The presence of otalgia should prompt evaluation for bullous myringitis. Bullous myringitis will resolve with antibiotics directed at the S. pneumoniae pneumonia. B. pertussis (A) is the causative organism for pertussis or whooping cough. H. influenzae (B) is common causes of pneumonia but do not cause bullous myringitis. Streptococcus bovis (C) is associated with endocarditis and colorectal cancer.

A 65-year-old woman presents to the ED for right-sided facial weakness that began acutely 1 hour prior to arrival. She woke up from sleep this morning with a dry right eye and is now unable to move the right side of her mouth. You ask the patient to smile and raise her eyebrows and you note a right-sided facial droop and no forehead wrinkling on the right side. Which of the following is the most appropriate next step in management? Administer a small dose of edrophonium, followed by a full dose, and observe the patient for improvement of her symptoms Administer intravenous prochlorperazine Obtain a blood glucose level, basic laboratory work, and CT scan of the head Obtain a National Institutes of Health Stroke Scale, bring the patient to the CT scanner, and consult a neurologist Provide prescriptions for corticosteroids, antivirals, artificial tears, and an eye patch, and have the patient follow up with her physician

Correct Answer ( E ) Explanation: This patient has Bell's palsy, a partial or complete paralysis of the facial nerve (cranial nerve VII). Typical symptoms include sudden onset of unilateral facial paralysis, including the forehead, decreased tearing, hyperacusis, and loss of taste sensation on the anterior two-thirds of the tongue that can progress over 1 to 7 days. Many patients describe a viral prodrome. The physical exam is key in the diagnosis to establish the presence of a peripheral rather than a central 7th nerve palsy. Upper and lower facial weakness is present in the peripheral nerve palsy. In central 7th nerve palsy, common in stroke syndromes, only the lower facial weakness is present. The forehead is spared because of bilateral hemispheric innervations to the forehead muscles. Treatment includes corticosteroids, antivirals, artificial tears, and eye protection. The National Institutes of Health Stroke Scale (D) helps to classify the severity of deficit in the setting of an acute CVA and should be obtained in all suspected cases. The patient in this vignette, however, had clear evidence of a peripheral 7th nerve palsy, so a stroke scale is not needed. Prochlorperazine (B) is often used in the treatment of migraine headaches but has no role in the management of Bell's palsy. Obtaining a blood glucose level (C) is important to rule out hypoglycemia or hyperglycemia as a cause of neurologic deficit and would be reasonable in this case, but further laboratory or radiographic evaluation is not necessary. Administration of edrophonium (A), a short-acting acetylcholine esterase inhibitor, is useful for suspected cases of myasthenia gravis (the Tensilon challenge test) and can help distinguish exacerbations from cholinergic crises. Although myasthenia gravis often involves facial (especially ocular) muscle weakness, systemic symptoms are not uncommon and, unlike Bell's palsy, deficits tend to worsen with activity.

A 19-year old man presents to the ED after rolling his left ankle playing basketball earlier in the evening. X-rays of the ankle do not reveal a fracture. The exam reveals a positive anterior drawer test with no endpoint on the left. Which of the following is the most likely diagnosis? Achilles' tendon rupture Grade 1 tear of the calcaneofibular ligament Grade 2 tear of the deltoid ligament Grade 3 tear of the anterior talofibular ligament

Correct Answer ( D ) Explanation: Based on the physical exam findings, a grade 3 tear of the anterior talofibular ligament (ATF) is the most likely diagnosis as the anterior drawer test is specific for the ATF and the lack of endpoint indicates a grade 3 or complete rupture of the ligament. The ATF is the most commonly injured ligament in an inversion sprain and is often the ligament that tears first. Inversion ankle sprains are classified by grade with grade 1 being the least severe and grade three being the most severe generally resulting in instability of the ankle and inability to bear weight. In addition to malleolar tenderness, the inability to bear weight if both at the time of injury and in the emergency department is an indication for X-ray via the Ottawa ankle rules. The physical exam of a patient with a ruptured Achilles' tendon (A) would have a positive Thompson's test in which the toes will not plantarflex when the calf muscles are squeezed. An injured deltoid ligament (C) is unlikely due to the mechanism of injury as deltoid ligament injuries generally occur from eversion of the ankle, not inversion. There would be valgus instability of the ankle with a deltoid ligament injury as well. A grade 1 tear (B) of the calcaneofibular ligament (CFL) would result in tenderness and mild swelling about the CFL about its origin on the anterior lateral malleolus to its insertion on the calcaneous. This is the second most commonly injured ligament, after the ATF, with an ankle sprain and would likely be injured with the degree of sprain above. It is evaluated with the talar tilt test with the foot in dorsiflexion and is severely injured if the ankle has increased inversion when the test is performed.

A patient presents with decreased inferior field vision. Fundoscopic examination is normal, however, external inspection shows a collection of blood in the anterior chamber of the eye. Which of the following is the most likely diagnosis? Hemianopia Hordeolum Hyperopia Hyphema

Correct Answer ( D ) Explanation: Hyphema is an accumulation of blood in the anterior chamber of the eye. They are frequently traumatic in nature, but also occur post intraocular surgery. Other causes include neovascularization, vascular abnormalities, eye tumors, uveitis, leukemia, hemophilia, anticoagulation and vonWillebrand disease. Immediate ophthalmologic consultation is required to decrease the chance of vision loss, and to evaluate intraocular pressure and optic atrophy. Treatment consists of analgesics, dilating drops, steroids, head of bed elevation, controlling intraocular pressure and nightly use of an eye patch. Hemianopia (A) is visual impairment or blindness in the left or right halves of the visual field. Most commonly due to stroke or traumatic brain injury, hemianopia is usually permanent. Hyperopia (C), also called farsightedness, describes the refractive condition of seeing clear distant objects but blurry proximal objects. A hordeolum (B) is an acute painful eyelid lesion occurring commonly on the lid margin or internal surface.

A man presents to the ED with angina during exertion that has increased in intensity and duration over the past 18 hours. An ECG shows T wave inversion in 4 leads. Which of the following risk stratification tools is used in the acute management of this type of angina? CHADS-2 Score Framingham 10-year Cardiovascular Disease Score Reynolds Score TIMI Score

Correct Answer ( D ) Explanation: Nearly 5 million Americans present to the emergency department with angina, with about 1.4 million of these patients getting admitted under the diagnosis of unstable angina (UA) or non ST elevation myocardial infarction (NSTEMI). Of the acute coronary syndromes, unstable angina (UA) and non ST-segment elevation myocardial infarction (NSTEMI) are similar conditions of subtotal coronary occlusion with myocardial ischemia or infarct, but both are distinctly different than ST-segment elevation myocardial infarction (STEMI) which represents complete coronary occlusion and myocardial infarct. UA is characterized by anginal episodes at rest or with minimal exertion that increase in intensity or frequency with time and are often accompanied by dynamic ECG changes. The initial evaluation of UA and NSTEMI includes risk stratification to predict the occurrence of adverse outcomes. One such guideline is the Thrombolysis in Myocardial Infarction (TIMI) risk tool. This seven point scoring tool utilizes historical facts, ECG findings and laboratory results to guide further therapeutic options. A higher score places the patient into a more invasive paradigm that usually includes cardiac catheterization, whereas a lower score favors a more conservative approach with medication options. The CHADS2 Score (A) is used to predict the likelihood of stroke in patients with atrial fibrillation, the results of which guide anticoagulant and antiplatelet therapy. The Framingham 10-year CVD (B) Score is a tool used to determine the likelihood of a patient having a heart attack in the next 10 years. It is not used in the acute evaluation of UA or NSTEMI. The Reynolds Score (C) is a tool used to predict a 10-year risk of MI, stroke or major heart disease. It too is not used in the acute evaluation of UA.

Absorption of vitamin B12 requires intrinsic factor. Which of the following cells of the stomach secretes intrinsic factor? Chief cells G-cells Mucous cells Parietal cells

Correct Answer ( D ) Explanation: Parietal cells are responsible for the secretion of intrinsic factor necessary for the absorption of vitamin B12. They also secrete hydrochloric acid (HCl) needed for digestion. Parietal cells are of particular importance in the pathology of pernicious anemia. One way pernicious anemia can develop is by loss of gastric parietal cells which are responsible for the secretion of intrinsic factor, a protein essential for subsequent absorption of vitamin B12 in the ileum. The autoimmune destruction of gastric parietal cells in pernicious anemia leads to a lack of intrinsic factor resulting in vitamin B12 deficiency. Chief cells (A) are responsible for the secretion of the proteolytic proenzymes pepsinogen I and II, as well as gastric lipase. Pepsinogen is activated into the digestive enzyme pepsin when it comes into contact with acid produced by the parietal cells. Pepsin degrades proteins into peptides. A simple way to remember the function of the chief cells is to think of the "Peppy Chief." G-cells (B) secrete gastrin, a peptide hormone that stimulates the secretion of hydrochloric acid by the parietal cells. Mucous cells (C) of the stomach, as the name implies, secretes mucous.

A healthy 7-year-old girl presents with the rash seen above. What management is indicated? Clotrimazole Dicloxacillin Meropenem Mupirocin

Correct Answer ( D ) Explanation: This child presents with a mild case of impetigo and can be treated with topical mupirocin. The rash is characterized by a slowly progressing pustular eruption and is commonly seen in preschool age children. The most common causative agent is Staphylococcus aureus with group A streptococcus as a less common etiology. It most commonly presents on the face and other exposed areas and typically begins with a single pustule that develops into multiple lesions over time. The original erythematous vesicle will break leaving red erosions covered in a golden yellow crust. The lesions should not be painful but may be pruritic. The lesions are contagious. First line treatment for limited extent of lesions is with topical antibiotics including mupirocin 2% ointment. Mupirocin should not be used if methicillin-resistant strains are suspected. Meropenem (C) is a broad spectrum antibiotic that can only be administered intravenously. Although it is effective against the bacteria that cause impetigo, this patient should be treated with an oral or topical agent. Clotrimazole (A) is an antifungal agent that would not be helpful in the treatment of impetigo. Dicloxacillin (B) is a semisynthetic penicillin that is indicated for the treatment of extensive bullous impetigo but not for pustular impetigo.

A 42-year-old woman who spent two days hospitalized after she underwent an appendectomy three weeks ago presents with cough, green sputum and fever. Her vitals are T 100.7°F, HR 94, BP 123/76, RR 18, and oxygen saturation 97%. She is well appearing and her blood work (CBC and BMP) is unremarkable. A chest X-ray shows a left lower lobe infiltrate. Which of the following represents the best management for this patient? Discharge home with oral antibiotics and follow up Draw blood cultures and discharge home on oral antibiotics Order a chest CT scan Start IV antibiotics and admit

Correct Answer ( D ) Explanation: This patient has a health-care associated pneumonia (HCAP) requiring IV antibiotics and admission. HCAP is defined as infection occurring within 90 days of a 2-day or longer hospitalization; in a nursing home or long-term care residence; within 30 days of receiving intravenous antibacterial therapy, chemotherapy, or wound care or after a hospital or hemodialysis clinic visit. HCAP requires IV broad spectrum antibiotics because it may involve both the typical pathogens involved in community-acquired pneumonia (CAP) (Mycoplasma pneumonia, Haemophilus influenzae, Streptococcus pneumoniae and Chlamydia pneumoniae) as well as more resistant organisms (Acinetobacter species, Pseudomonas species, Staphylococcus aureus (including MRSA) Enterobacter species, Escherichia coli, Proteus species, Klebsiella species etc.). Treatment should be as follows (one antibiotic from each category): This patient cannot be sent home (A & B) as she needs IV antibiotic therapy. A chest CT scan (C) can be useful in patients with a history of recurrent or non-resolving pneumonia or in those with high-risk features (i.e. prolonged smoking) but is typically not indicated for a first episode of pneumonia.

A patient is found to have the following basic metabolic panel results: sodium 143 mmol/L, potassium 3.6 mmol/L, chloride 107 mmol/L, bicarbonate 14 mmol/L, BUN 16 mg/dL, creatinine 1.2 mg/dL, and glucose 92 mg/dL. Which of the following is a possible cause of her acidosis? Addison's disease Diarrhea Renal tubular acidosis Salicylate toxicity

Correct Answer ( D ) Explanation: This patient has a metabolic acidosis as defined by a decrease in serum bicarbonate concentration. This can occur when there is the addition of excess acids, when acid secretion is impaired, or when there is an inappropriate loss of alkali. The causes of metabolic acidosis can be divided into elevated anion gap and normal anion gap. The anion gap is calculated using the formula Na - (Cl + HCO3) and a normal anion gap is 10 +/- 2. In this example, the patient's anion gap is 22 (143 - (107 + 14)) and therefore considered elevated. Causes of an elevated anion gap acidosis, defined as an anion gap > 12, can be remembered by using the mnemonic MUDPILES (Methanol, Uremia, Diabetic ketoacidosis, Paraldehyde, Isoniazid, Lactic acidosis, Ethylene glycol, and Salicylate toxicity). Addison's disease (A), diarrhea (B), and renal tubular acidosis (C) are all causes of a normal anion gap metabolic acidosis.

A 25-year-old G2P1 presents to your office at 32 weeks gestation with a complaint of severe itching, particularly on the palms of her hands and the soles of her feet. Lab results reveal elevated bile acids. Regarding this disease, which of the following statements is most correct? Aminotransferases are low Disease recurrence is rare in subsequent pregnancies The treatment of choice is cholestyramine There is an increased risk for fetal demise

Correct Answer ( D ) Explanation: This woman has classic symptoms of intrahepatic cholestasis of pregnancy. It is characterized by pruritus which is often concentrated in the palms of the hands and soles of the feet. Serum bile acids are almost always elevated and there is a significant increase in intrauterine fetal demise. The pathophysiology of fetal death is poorly understood, but may be related to fetal dysrhythmias or placental vasospasms from high bile acid levels. There is no ideal antepartum testing strategy, but regular non stress tests and biophysical profiles looking for signs of fetal compromise are often performed. A widely accepted management approach is induction of labor between 36 and 37 weeks gestation as most fetal deaths occur after 37 weeks. Aminotransferases are often elevated (A) in this disease. The treatment of choice is ursodiol which helps increase hepatic bile flow and decrease bile acid levels. This improves pruritus and also helps lower aminotransferase levels. Cholestyramine (C), which decreases absorption of bile salts in the ileum, is not as effective as ursodiol in decreasing pruritus. Intrahepatic cholestasis of pregnancy will recur (B) in over half of subsequent pregnancies.

A 23-year-old Caucasian woman presents to her primary care provider complaining of an unsightly mole on her left calf that has enlarged over the past 6 months. Physical exam reveals a 7 mm dark asymmetric macule with irregular borders. Which of the following is the most likely diagnosis? Acral lentiginous melanoma Lentigo melanoma Nodular melanoma Superficial spreading melanoma

Correct Answer ( D ) Explanation: This woman most likely has superficial spreading melanoma, the most common subtype of melanoma. Melanoma is a malignant tumor formed by melanocyte transformation. Melanoma is the sixth most common cancer in the United States. Cutaneous melanoma is divided into four major subtypes: superficial spreading, nodular melanoma, lentigo melanoma, and acral lentiginous. Risk factors for melanoma include ultraviolet radiation, history of sunburns, tanning beds, psoralen and ultraviolet A radiation (PUVA) therapy, increased number of nevi, history of melanoma, immunosuppression, increasing age, and fair skin. Key features of melanoma that should prompt referral to a dermatologist are lesion asymmetry, irregular borders, color variations, diameter > 6 mm, and elevated surface. Lesion growth or change, crusting or bleeding, or inflammation should also raise suspicion for melanoma. When performing a skin exam, it is important to not only examine the entire body surface, but to also examine the scalp, palms, soles, digits, and nails. High risk patients should be instructed to perform skin self-examinations. Patients with suspicious lesions should be referred to a dermatologist for biopsy. Depth of invasion is the single most important determinant of prognosis. As tumor thickness increases, survival rate decreases. Acral lentiginous melanoma (A) is the most common type of melanoma in African Americans, however it accounts for less than 5% of all melanomas. Acral lentiginous melanoma are most commonly found on the palms, soles, and underneath the nails. Subungual melanomas appear as longitudinal brown or black bands or masses. Lentigo melanoma (B) is most commonly seen in older adults. Lentigo melanoma is usually found on sun-exposed areas such as the scalp, ears, face, and neck. Lentigo melanoma accounts for approximately 10-15% of all melanoma cases. Nodular melanoma (C) appear as darkly colored, pedunculated, or polypoid nodules. Nodular melanoma is the second most common subtype of melanoma. Based on this patient's race, lesion description, and lesion location, superficial spreading melanoma is more likely subtype of melanoma.


Ensembles d'études connexes

The 7 Ideas that Shook the Universe Final Exam Part 2 of 3

View Set

CHEMISTRY- exam (old test questions)

View Set

Chap 1 problem set August 2016 Xu Microecon GCSU

View Set

Real Estate Exam Prep Chapter 3: Property valuation and financial analysis

View Set

Perry: Chapter 4: Reproduction Systems Concerns EAQ

View Set

Comparing fractions, Decimals, and Percents

View Set